True or False: It is possible to find all the hormones in a blood sample taken from the arm.
True
True or False: Steroids exert their effects by increasing the amount of cAMP in the target cells.
False
Amino acid derived hormones (amines) exert their effects by increasing the amount of cAMP in the target cells.
True or False: Hormones do not stimulate all cell types with equal probability.
True
True or False: All hormones secreted by the pituitary actually came from the hypothalamus.
False
Only hormones secreted by the posterior pituitary came from hypothalamus.
True or False: Diabetes insipidus results from an excess production of ADH.
False
Diabetes insipidus results from an inadequate production of ADH.
True or False: High levels of TSH would suppress thyroxine by negative feedback.
False
High levels of thyroxine would suppress TSH by negative feedback.
Choose the statement about steroid hormones that is incorrect.
A) Steroids employ first and second messengers that amplify the cellular response.
Amines employ first and second messengers that amplify the cellular response.
Prostaglandins are ___________ substances that regulate neighboring cells.
A) Paracrine
Paracrine hormones act locally by diffusing from their source to target cells in the neighborhood.
Endocrine hormones are distributed in blood and bind to distant target cells.
Autocrine hormones act on the same cell that produced them.
Cyclic AMP functions as a(n) _______________ for _______________ hormones.
D) Secondary messenger; nonsteroid
Which of these is not a method the body uses to regulate its hormonal releases?
A) Degradation of the endocrine gland
Where are the posterior pituitary hormones manufactured?
D) In neurosecretory cells that originate from the hypothalamus
There are several types of cells in the anterior pituitary. Which ones secrete growth hormone?
A) Somatotropes
Which one of these anterior pituitary hormones shows increased secretion in response to stress?
C) Adrenocorticotropic hormone
Drinking alcoholic beverages on hot days is not safe because alcohol inhibits the release of ____________ which normally helps to conserve water during dehydration.
D) Anti-diuretic hormone
The thyroid hormones are responsible for regulating metabolic rate and calcium and phosphate ion concentration in the blood. Which thyroid hormone lowers calcium and phosphate ion concentration in the blood?
D) Calcitonin
Hypothyroidism in infants can result in _______________.
C) Cretinism
Congenital hypothyroidism, previously known as cretinism, is a severe deficiency of thyroid hormone in newborns.
The hormone _______________, which is secreted by the adrenal _______________, causes the kidney to conserve sodium and excrete potassium ions and indirectly helps to maintain systemic blood pressure.
A) Aldosterone; cortex
Sex hormones are secreted by:
D) Inner cortex of adrenal medulla and gonads
In the pancreas, ______________ cells secrete insulin, which ____________ blood levels of glucose.
B) Beta; lowers
The alpha cells of pancreas secrete glucagon, which raises blood levels of glucose. The delta cells secrete somatostatin, which inhibit the actions of insulin and glucagon.
What seems to be the cause of juvenile-onset or insulin-dependent diabetes mellitus (IDDM)?
A) Immune cells attack the pancreas that can then no longer produce insulin.
In adult-onset or insulin-independent diabetes mellitus, the receptors on the target cells become no longer responsive to insulin.
The endocrine gland responsible for the body's circadian rhythm is the ______________ gland.
D) Pineal
The pineal gland secretes melatonin, which is a hormone that helps regulate circadian rhythms. Melatonin is produced according to the amount of light a person is exposed to. The pineal gland releases greater amounts of melatonin when it is dark, which points to melatonin's role in sleep.
Which of these is not an endocrine property?
C) Rapid acting effects
Responses to endocrine system stimulation are typically slow but long lasting.
Which of these does not belong with the others?
A) Steroid
In the "second messenger" theory, which is the first messenger?
B) Enzyme
First messengers are the extracellular substances that can initiate intracellular activities, while second messengers are the intracellular signalling molecules that send signals from receptors to targets within the cell.
Which of these is not an endocrine gland?
B) Salivary gland
Salivary gland is an exocrine gland.
Which is not a function of the hypothalamus?
B) Secrete FSH
The anterior pituitary secretes the FSH.
Which of these hormones is made by the posterior pituitary?
B) ADH
Which hormone directly affects blood pressure?
C) ADH
Anti-diuretic hormone may increase the blood pressure by promoting reabsorption of water in the distal tubules of the nephrons.
Which hormone causes the contractions of labor.
D) Oxytocin
Oxytocin contracts the uterus and promotes the progress of labour.
Which hormone has the greatest influence on the ability to secrete milk?
C) Prolactin
Prolactin is necessary for the secretion of milk by the cells of the alveoli. The level of prolactin in the blood increases markedly during pregnancy, and stimulates the growth and development of the mammary tissue, in preparation for the production of milk.
Which hormone causes excessive skeletal growth or gigantism?
D) Somatotropin
Somatotropin is another name for growth hormone.
Which of these does not directly stimulate the gonads?
D) TSH
Interstitial cell stimulating hormone (ICHS) is produced by the anterior lobe of the pituitary gland that, in the male, stimulates the interstitial cells of the testes to produce testosterone. It is chemically identical to luteinizing hormone of the female.
Which hormone has the greatest effect on metabolism?
A) Thyroxine
True or False: The cells that are stimulated by hormones are referred to as its receptors.
False
Hormones bind to areas on cells called receptors, and the cells are referred to as specific hormone targets.
True or False: An organ can belong to the digestive and endocrine systems simultaneously.
True
There are areas in the digestive and other systems that secrete hormones and are therefore endocrine in nature. Gastrin is a hormone secreted by the stomach.
True or False: Hormones secreted by the adrenal gland can have direct neurological effects.
True
The adrenal medulla produces the amines called catecholamines such as adrenalin which effect targets in the same manner as the adrenalin compounds produced by axons of the sympathetic nervous system.
True or False: Oxytocin is a protein secreted by the posterior pituitary.
False
Oxytocin is a peptide produced by the posterior portion of the pituitary gland.
True or False: Thyroxine is a thyroid hormone also called T3.
False
Thyroxine contains four iodine atoms and is abbreviated T4. T3 is triiodothyronine.
True or False: Hormones exert their effects by causing organs to perform functions they could not otherwise perform.
False
Although hormones have potent effects on target cells, they cannot normally cause a cell to do anything it could not genetically have the ability to do. Hormones alter metabolic function.
True or False: Steroid hormones have their effect on the nucleus and genes of a target cell.
True
Steroid hormones enter a cell and cause the DNA of the nucleus to express specific genes that will instruct the RNA to synthesize proteins.
True or False: Non-steroid hormones are called second messengers because they influence the binding site first and then the activity site second.
False
The second messenger is usually cAMP that is stimulated by the non-steroid hormones that complex with the binding sites on the cell membrane.
True or False: Various protein kinases are enzymes that stimulate cells to increase their metabolic activities.
True
In the second messenger theory, hormones can increase the metabolic rates and functions of cells by causing cAMP to eventually result in the activation of kinase enzymes that will alter the structure and function of proteins.
True or False: Cyclic guanosine monophosphate is an enzyme which causes the target effects seen with some hormones.
False
cGMP is similar to cAMP and functions as a second messenger with some hormones. cGMP is also a nucleotide.
True or False: Prostaglandins are considered hormones, which are stored within cells and released on demand.
False
Prostaglandins are sometimes not considered true hormones and they are not stored within cells. Since they influence cAMP production, they can be considered hormone mediators.
True or False: Prostaglandins are known to cause the smooth muscle of the uterus or intestines to contract.
True
Prostaglandins are the mediators that can cause premenstrual cramping or the cramping associated with intestinal colitis.
True or False: The levels of insulin in the blood are maintained at constant, stable, homeostatic levels.
False
The physiologic parameters such as blood glucose are maintained by negative feedback mechanisms involving several hormones. Hormones such as insulin are not maintained but produced on demand.
True or False: The pituitary and adrenal gland can be controlled by the direct influence of the nervous system.
True
The posterior pituitary is directly controlled by the hypothalamus of the brain and the adrenal medulla responds to sympathetic nervous system stimulation.
True or False: The pituitary is located within the sella turcica of the ethmoid bone, and is attached to the thalamus by the infundibulum.
False
The pituitary is located in the sella turcica of thesphenoid bone and is attached to the hypothalamus by its stalk, the infundibulum.
True or False: As blood levels of thyroid hormones increase, the secretions of TRH and TSH are reduced.
True
Thyroid hormones such as thyroxine reduce the secretion of thyroid releasing hormone and thyroid stimulating hormone through negative feedback.
True or False: Alcohol is known to be a potent anti-antidiuretic.
True
Ethyl alcohol causes an inhibition of ADH production and consequent increase in urine output.
True or False: The hypothalamus of the brain has osmoreceptors, which are sensitive to changes in blood concentration.
True
The osmoreceptors respond to increases in concentration by increasing the release of ADH which will retain water in an attempt to dilute and return the concentration pressure to normal.
True or False: Diabetes insipidus is a condition that results in edema and swelling because of an excess of ADH.
False
Diabetes insipidus usually results in a drastic reduction in ADH production with a consequent increase in urine output and dehydration.
True or False: Cretinism is a condition of physical and mental retardation that is the result of a deficiency of thyroxine.
True
Deficiency of the thyroid hormones in the newborn or pregnant mother will result in a diminished ability to thrive and develop.
True or False: It is possible to have high levels of TSH but a decrease in the levels of thyroxine.
True
Normally thyroxine will inhibit the production of TSH by negative feedback, but in simple or endemic goiter, iodine and thyroxine are lacking.
True or False: Thyroxine production can be increased by TSH or antibodies.
True
In Grave's disease, autoantibodies are produced that bind to thyroid receptors and mimic the effects of TSH.
True or False: A pheochromocytoma is a tumor of the adrenal cortex which raises blood pressure.
False
The adrenal medulla can have a tumor that produces catecholamine neurotransmitters that mimic the effect of the sympathetic nervous system.
True or False: Male sex hormones can be produced by the testes or adrenal glands.
True
Androgens such as testosterone are synthesized in the testes and adrenal cortices of both sexes.
True or False: Cortisone can either cause or inhibit inflammation.
True
Cortisone can cause inflammation at low physiologic levels and block inflammation when the dose levels are high or therapeutic. Recall that inflammation is a normal defense mechanism.
True or False: The pancreas is an endocrine and exocrine gland that secretes insulin from the alpha cells.
False
The pancreas has a digestive exocrine function and also produces insulin from its beta cells.
True or False: Cancer cells can produce hormones that affect blood sugar and blood calcium levels.
True
Cancer cells can secrete an insulin-like substance, PTH, ACTH or ADH.
True or False: Circadian rhythms are those that involve fluctuations that usually last several days.
False
Circadian cycles are biorhythms that repeat every twenty four hours or have day or night fluctuations such as the sleep wake/cycle.
The two systems that act to control all body activities are the nervous and _____________ systems.
C) Endocrine
All body functions are influenced in some way by the fast-acting nervous system and relatively slow responding hormones.
The ______________ system includes the glands that release their secretions directly into the blood.
B) Endocrine
The exocrine system has glands with ducts that open to external surfaces. The endocrine glands produce hormones that are secreted directly into the capillaries of each gland.
Which of the following glands does not belong to the endocrine system?
A) Salivary
The salivary glands have ducts that open onto the surface of the oral cavity and are therefore exocrine.
Which of the following is not a chemical class of hormones?
A) Carbohydrate
Some hormones such as FSH contain a carbohydrate portion but are glycoproteins. No hormone is a carbohydrate by itself.
Which of these is not a steroid?
C) Adrenalin
Adrenalin has a NH2 group and is classed as an amine (catecholamine).
Which of the following is the precursor that will result in the synthesis of the others?
C) Cholesterol
All of the steroids including vitamin D are in equilibrium with their dietary precursor cholesterol. Cholesterol by itself is not considered a hormone.
Which of the following does not belong with the others?
A) Cortisone
Cortisone is a steroid and the adrenalin (epinephrine) derivatives are amines.
Which of the following does not chemically belong with the other hormones?
C) ADH
: Although all of these are comprised of amino acids, short chains are referred to as peptides, such as antidiuretic hormone, and more complex structures are classed as proteins.
Which of these is not produced by the anterior pituitary?
C) Somatostatin
Somatostatin is a growth hormone made by the hypothalamus. Somatotropin is another name for anterior pituitary growth hormone.
Which of these does not belong with the others?
A) Follitropin
Follitropin is another name for follicle stimulating hormone (FSH). The others refer to luteinizing hormone.
Which of the following is not a pancreatic hormone?
D) Aldosterone
Aldosterone is a steroid produced by the adrenal cortices.
Which of these compounds is produced first?
A) Adenylate cyclase
Adenylate cyclase is the enzyme caused to be produced by the hormone. This enzyme then catalyzes the conversion of ATP into cyclic adenosine monophosphate which then stimulates various enzymes called kinases which alter proteins.
Which factor inactivates and decreases the levels of cAMP?
B) Phosphodiesterase
Phosphodiesterase is an enzyme which inactivates cAMP almost as soon as it is formed. This is one mechanism that accounts for the cyclic timing of hormone-mediated reactions.
Which of the following does not employ cAMP second messengers?
D) Cortisol
Cortisol is a steroid hormone that functions by way of DNA and genetic mechanisms.
Which of the following is not a hormone function?
D) Convert fibroblasts to osteoblasts
Hormones usually do not play a major, direct role in causing one cell type to convert to another. Functions such as bone development are influenced by hormones as well many other complex dietary and genetic factors.
Prostaglandins are ___________ that affect various functions.
C) Lipids
Although proteins are the usual functional molecules in biochemistry, there are lipids such as the steroids and prostaglandins that have powerful effects on cell activities.
The precursor molecule that produces all of the prostaglandins is ________________.
D) Arachidonic acid
Arachidonic acid is the lipid that can be converted into the prostaglandins. Arachidonic acid is one of the metabolically significant fatty acids.
Which factor is a direct cause of the rheumatoid arthritis symptoms?
D) Prostaglandin
There are several types of prostaglandins which can cause inflammation of joints or related structures. Cortisone is used as an anti-inflammatory drug.
Which of the following is not a property of prostaglandins?
C) All of these are functions
Prostaglandins are mediators in almost all physiologic functions, normal as well as the abnormal disease alterations.
Which of these is an example of negative feedback?
D) Low blood sugar inhibits insulin.
Negative feedback occurs whenever a parameter such as sugar causes a hormone to cease or reduce its activity. The others are examples of positive feedback.
Releasing hormones are produced by the:
A) Hypothalamus
The releasing hormones are chemical factors that are secreted from the hypothalamus into the pituitary in order to control the release of the pituitary hormones.
Which hormone controls the ability to produce milk?
B) PRL
Prolactin (PRL) stimulates milk glands.
Where are releasing hormones secreted into?
C) Hypophyseal portal veins
The hypophyseal portal vein is a direct connection or shunt between the hypothalamus and anterior lobe of the pituitary.
Which of the following directly influences or controls the other areas?
D) Hypothalamus
The hypothalamus controls the pituitary. These structures, pituitary and hypothalamus, affect more areas in the body than any other endocrine gland.
Which of the following is an incorrect association?
D) Thyrotropes; thyroxine
The thyrotropes are cells within the anterior pituitary that secretes TSH which causes the thyroid to secrete thyroxine.
Which of the following directly stimulates mitosis?
A) Somatotropin
Somatotropin or STH or GH, causes increases in the size of the body by stimulating cell division and anabolic reactions.
GHRH stimulates the release of GH.
Which of these will decrease the release of growth hormone?
B) High protein diet
High levels of protein in the blood can inhibit GH by a negative feedback mechanism. Although this seems ironic, low protein levels will actually increase the release of GH.
The cartilage of bones is stimulated by:
C) Somatomedin
Somatomedin is a mediator secreted by the liver in response to GH. Somatomedin is needed to stimulate cartilage growth.
The condition called _______________ occurs in adulthood and is evidenced by thickened skull bones and distortions in the arms and hands.
C) Acromegaly
Because of closure of the bone epiphyses, excess GH cannot result in gigantism if it is excess in an adult, but will cause deformities.
Which hormone can cause sterility in a male?
D) PRL
Prolactin (PRL) can cause decreases in LH or ICSH levels which can in turn result in lower levels of testosterone.
The hormone _____ causes the breast to increase its synthesis of milk.
A) PRL
Prolactin (PRL) stimulates the secretion of milk. PRL is stimulated by prolactine releasing factor or hormone (PRF) and inhibited by prolactin release inhibiting factor or hormone (PIF).
Which of the following is not produced by the pituitary?
D) GnRH
GnRH is gonadotropin releasing hormone produced by the hypothalamus.
Which hormone decreases the production of urine?
A) ADH
Antidiuretic hormone increases water absorption and reduces urine output.
Oxytocin is most closely related chemically to:
B) Anti-diuretic hormone
Both hormones are peptides comprised of nine amino acids. ADH contains phenylalanine and arginine in place of the isoleucine and leucine in oxytocin.
ADH has the ability to increase blood pressure by constricting blood vessels and is therefore also called ______________.
B) Vasopressin
Vasopressin is another name for ADH and acts as a vasoconstrictor as well as water-retaining hormone.
Which hormone plays the greatest role in the contractions that occur in the uterus during labor?
B) Oxytocin
Oxytocin has a powerful effect on smooth muscle contraction and is sometimes given to induce labor. Prostaglandins also constrict uterine muscle.
Which hormone is most closely involved with ovulation?
B) LH
Luteinizing hormone causes the follicle to release its egg cell in ovulation. FSH is more important in the early development of the follicles.
The ovary and __________ contain spherical collections of cells referred to as follicles.
C) Thyroid
The thyroid follicles produce the glycoprotein aggregates called the colloid which will result in thyroxine synthesis.
Which hormone exerts the greatest influence on metabolism?
D) Thyroxine
Thyroxine, T4, and triiodothyronine T3, increase the metabolic rates of most cells.
Which of the following does not belong with the others?
D) Calcium
The thyroxine molecule requires iodine and the amino acid tyrosine as parts of its structure.
Which of the following is not true of thyroxine?
B) Decreases protein synthesis
Thyroxine increases the breakdown of lipids but promotes protein synthesis. Glucose oxidation is enhanced.
Which of the following have the most potent effect on increasing metabolic rate?
B) Triiodothyronine
T3 is five times more potent than T4. Thyroglobulin is the storage form of the thyroxine hormones which are transported in the blood in an active form by alpha globulin proteins.
Which of the following is not true of calcitonin?
A) Increases blood calcium
If acting by itself, calcitonin would have the effect of decreasing blood calcium levels and increasing bone calcium deposition.
Which of the following is not a symptom of hyperthyroidism?
A) Increase in ability to sleep
Thyroxine increases metabolic rate and can cause a state of restlessness. Intellectual abilities are actually enhanced.
Graves disease can cause all of these except which one?
C) General edema
Thyrotoxicosis can cause post-orbital edema and is probably related to autoimmune factors. General edema is a symptom of myxedema or hypothyroidism.
Which gland is assessed by the use of radioactive iodine?
A) Thyroid
The thyroxine hormones, T4 and T3 will incorporate the radioactive iodine the same way they would normal iodine, which then can be traced by radiation counter devices.
The ____________ directly antagonizes the effect of calcitonin.
C) Parathormone
Parathormone (parathyroid hormone) acts to increase blood levels of calcium and bone resorption which is the opposite of calcitonin effects.
What is the most active form of vitamin D?
C) Dihydroxycholecalciferol
Vitamin D originates from the steroid cholesterol which will undergo several conversions before becoming active (D3) .
What is the final factor in the synthesis of the active form of vitamin D?
C) Kidney function
Cholesterol conversion progresses from the skin to the intestines and finally reaches completion in the kidneys as hormonally vitamin D3.
Which of the following is not consistent with a diagnosis of hyperparathyroidism?
C) Muscle cramps
Muscle cramping is usually the result of a deficiency of PTH or hypoparathyroidism in which blood calcium levels are suppressed.
Osteoporosis most likely results from:
C) Increased PTH secretion
Osteitis fibrosa is the condition of fibrous deposition in the cavities left by the resorption of calcium in osteoporosis. Excesses in PTH will stimulate the osteoclasts to dissolve bone calcium salts.
Which of the following is the last step in the sequence of hormone synthesis in the adrenal medulla?
D) Epinephrine
The medulla amines are derived from the amino acids phenylalanine and tyrosine. The adrenalin compounds are therefore closely related and easily inter-converted.
Which of the following is not true of aldosterone?
D) Decreases blood Na+ levels
Aldosterone conserves sodium in exchange for potassium and has a potent effect on blood volume.
Which of the following is the factor that initiates the other steps?
B) Renin
Renin is released from the juxta-glomerular cells of the kidney and converts a blood protein called angiotensinogen to angiotensin I.
Which of these is not true of cortisol?
A) Decreases blood glucose
Cortisol has a general effect of increasing blood sugar levels by affecting the liver and various other tissues.
The following relate to Cushing's syndrome except which one?
D) Undersecretion of ACTH
Cushing's syndrome patients sometimes have characteristic fat depositions in the face and posterior lumbar areas along with hypertension and high blood sugar.
The following are true of insulin except which one?
A) Can cause hyperglycemia
Insulin lowers blood glucose and could cause hypoglycemia if in excess.
In NIDDM, the patient is usually over forty and has normal levels of ___________.
C) Insulin
In non-insulin-dependent diabetes mellitus, the levels of insulin are usually not suppressed as they are in IDDM or type I.
The hormone melatonin is secreted from the __________ gland.
B) Pineal
Melatonin from the pineal gland in the brain, is derived from serotonin and affects female reproductive cycles.
The ___________ gland is important for producing immunity.
D) Thymus
The thymus gland produces thymosin and other factors which account for the activity of the T cell lymphocytes.
Which gland produces a hormone, which stimulates the production of blood cells?
C) Kidney
The kidney secretes erythropoietin which stimulates blood cell formation in periods of need or stress.
In the general stress syndrome, all of the following are increased except which one?
A) Digestion
Stress tends to increase sympathetic nervous arousal which is antagonistic to normal digestion and which could result in stomach ulcers or other gastrointestinal disorders such as diarrhea.
In which of the following locations does the final enzymatic step of vitamin D activation (the hydroxylation of the first carbon residue of vitamin D) occur?
B) Kidney
The final enzymatic step in the activation of vitamin D (the hydroxylation of the first carbon residue of vitamin D) occurs in the kidneys. This is because the kidneys express the enzyme that catalyses this reaction: 1-alpha-hydroxylase. This explains why kidney failure can lead to hypocalcemia and consequent bone disease.Dietary vitamin D is absorbed by the small intestine, not activated by it. Once absorbed, it travels in plasma, but is not activated there either.
Parathyroid hormone secreted by the parathyroid glands stimulates the hydroxylation of the first carbon residue of vitamin D in the kidneys by activating 1-alpha-hydroxylase.
Hydroxylation of the first carbon residue of vitamin D does not occur in the bones.
Which type of molecule is insulin?
B) Protein
Insulin is a protein. This explains why it must be given subcutaneously; if it is swallowed, it will be broken down by protein-degrading enzymes in the gastrointestinal tract, like any protein.
Where are the parathyroid glands located?
A) On the posterior surface of the thyroid gland
The parathyroid glands are located on the posterior surface of the thyroid gland. They are paired glands, with 2 on the posterior surface of the right lobe of the thyroid gland and 2 on the posterior surface of the left lobe. They secrete parathyroid hormone, which increases plasma calcium levels.
This explains why surgical removal of the thyroid gland or destruction of the gland with radioactive iodine can lead to the complication of hypocalcemia.
Which hormone is released by the beta-cells of the endocrine pancreas?
C) Insulin
Insulin is the hormone released by the beta-cells of the endocrine pancreas. This explains why the destruction of the beta-cells leads to type 1 diabetes mellitus, a condition in which there is an absolute insulin deficiency.
Although all of the other options are hormones produced by the endocrine pancreas, none of them are produced by beta-cells.
Ghrelin is released by the epsilon-cells; glucagon is released by the alpha-cells; pancreatic polypeptide is released by the F cells; and somatostatin is released by the delta-cells.
Which of the following hormones opposes the effect of insulin?
A) Glucagon
Glucagon opposes the effect of insulin. Glucagon raises blood sugar levels, whereas insulin lowers them. This explains why intramuscular glucagon injection can be used to treat those who have lost consciousness due to hypoglycemia as a result of exogenous insulin administration.
Amylin, gastric inhibitory peptide, glucagon-like peptide-1 and glucagon-like peptide-2 all enhance the effect of insulin.
Which of the following ion channels is involved in insulin secretion by beta-cells of the endocrine pancreas?
B) ATP-dependent potassium channel
The ATP-dependent potassium channel is involved in insulin secretion by the beta-cells of the pancreas. When glucose enters beta-cells, it is metabolised to pyruvate by glycolysis, which produces adenosine triphosphate (ATP). This ATP then binds to and closes the ATP-dependent potassium channel, which allows potassium out of beta-cells when open. The resulting build-up of potassium inside beta-cells causes depolarisation, which leads to calcium influx. This rise in intracellular calcium is the signal that triggers vesicles containing insulin to be released from beta-cells.
This explains why a class of drugs that close this channel (called sulphonylureas) is used to increase insulin secretion in those with type 2 diabetes mellitus.
Which of the following is a product of proinsulin that can be used to quantify endogenous insulin secretion when measured in the blood?
C) C-peptide
C-peptide is a product of proinsulin that can be used to quantify insulin secretion when measured in the blood. This is because the amount of C-peptide secreted from beta-cells of the endocrine pancreas is equal to the amount of insulin secreted by them and C-peptide is not metabolised by the liver, whereas insulin is. Measuring the amount of C-peptide in the blood is therefore important clinically to distinguish between type 1 and type 2 diabetes mellitus; in type 1, C-peptide levels will be low, whereas they will be normal in type 2.
Preproinsulin is the initial protein product of the insulin gene. Proinsulin is produced from it by enzymatic cleavage in the rough endoplasmic reticulum.
Enzymatic cleavage of proinsulin then yields 2 products: insulin and C-peptide. Insulin is made up of 2 polypeptide chains (an alpha-chain and a beta-chain) joined by disulphide bonds.
Which of the following is an effect of parathyroid hormone?
E) Increased reabsorption of calcium by kidneys
Parathyroid hormone, secreted by the parathyroid glands, has the net effect of increasing plasma calcium levels. One of the mechanisms by which it does so is by increasing calcium reabsorption by the kidneys. Another of its effects is reduced reabsorption of phosphate by the kidneys. Both of these mechanisms explain why people with kidney failure develop hypocalcemia and hyperphosphatemia. In kidney failure, the impaired reabsorption of calcium by the kidney stimulates parathyroid hormone release. However, since this hormone cannot increase calcium reabsorption and reduce phosphate reabsorption by the kidneys due to kidney failure, hypocalcemia and hyperphosphatemia develop.
Parathyroid hormone also increases plasma calcium levels by inhibiting bone formation by osteoblasts and stimulating bone resorption by osteoclasts.
Parathyroid hormone does not play a role in stimulating hydroxylation of the 25th carbon residue of vitamin D. The liver carries this out, which explains why liver disease can lead to hypocalcemia.
Which of the following converts 7-dehydrocholesterol (vitamin D1) into cholecalciferol (vitamin D3)?
B) Ultraviolet radiation
Ultraviolet radiation in sunlight converts the cholesterol derivative 7-dehydrocholesterol (vitamin D1) on the skin into cholecalciferol (vitamin D3). This explains why those with vitamin D deficiency are encouraged to expose their skin to sunlight and why those who live in countries with low sunlight levels often have suboptimal vitamin D levels.
Once cholecalciferol is formed, 2 hydroxyl groups are enzymatically added to its structure that greatly increase its hormonal activity. The first is the hydroxylation of the 25th carbon residue of cholecalciferol by the enzyme 25-alpha-hydroxylase in the liver, forming 25-hydroxycholecalciferol. The first carbon residue of 25-hydroxycholecalciferol is then hydroxylated by the enzyme 1-alpha-hydroxylase in the kidneys, forming 1,25-dihydroxycholecalciferol, the most hormonally active form of cholecalciferol. Parathyroid hormone stimulates this second hydroxylation reaction by activating 1-alpha-hydroxylase.
Calcitonin, secreted by the parafollicular cells of the thyroid gland, does not play a role in vitamin D metabolism. It also has the opposite effect to vitamin D; vitamin D increases plasma calcium levels, whereas calcitonin reduces them.
Which artery supplies the superior thyroid gland?
B) Superior thyroid artery
The superior thyroid artery supplies the superior portion of the thyroid gland. This is a branch of the external carotid artery, just before the cornu of the greater horn of the hyoid bone. This artery must be ligated during thyroid surgery. If cut and not ligated or ligated incompletely, it will bleed profusely and can lead to the postoperative complication of an expanding neck haematoma.
It doesn't just supply the superior portion of the thyroid, though, it also gives rise to:
1. Hyoid artery
2. Sternocleidomastoid artery
3. Superior laryngeal artery
4. Cricothyroid artery
The inferior portion of the thyroid gland is supplied by the inferior thyroid artery, a branch of the thyrocervical trunk.
You observe the following set of results of a recent water deprivation test:
Urine Osmolality (mOsm/kg)
After fluid deprivation
950
After desmopressin
570
What will you conclude from these results?
E) The results are incorrect.
The patient has a normal urine osmolality after fluid deprivation (>850 mOsm/kg). This is due to endogenous antidiuretic hormone (ADH). After administration of desmopressin, which is exogenous ADH, urine osmolality should either rise or stay the same depending on the underlying pathology. These results show a physiologically impossible (or, at least, an extremely unusual) response to desmopressin, and can therefore be concluded to be incorrect results.
Primary polydipsia would result in high urine osmolality after both fluid deprivation and desmopressin. The patient would have low urine osmolality before both tests due to excessive water consumption.
Nephrogenic diabetes insipidus would result in low urine osmolality after both fluid deprivation and desmopressin because the issue is not due to the absence of ADH.
Neurogenic diabetes insipidus would result in low urine osmolality after fluid deprivation and a relatively elevated urine osmolality after desmopressin because the absence of ADH is corrected with the administration of desmopressin.
SIADH would result in high urine osmolality after both fluid deprivation and desmopressin. The patient would have high osmolality before both tests due to higher circulating levels of antidiuretic hormone.
A 45-year-old woman presents to her GP complaining of irritability. She also mentions he has lost weight recently and regularly has to change her shirt during the work day because she sweats profusely. Thyroid Functions tests are performed which show:
TSH: 12 mU/L (0.4-4.0 mU/L)
T4: 32 mU/L (9-24 pmol/L)
Which single additional symptom is this woman most likely to admit to when questioned?
E) Difficulty driving due to reduced peripheral vision
This lady has secondary hyperthyroidism, indicated by both a high TSH and a high T4. The problem could either be a hypothalamus secreting too much thyroid releasing hormone (TRH) or a functional pituitary adenoma secreting too much TSH.
The correct answer is difficulty driving due to reduced peripheral vision - this might indicate bi-temporal hemianopia due to compression of the optic chiasm by a large TSH secreting pituitary tumour (macroadenoma).
The other options are incorrect, as described below.
Increase in shoe size - This suggests acromegaly, which is the result of a pituitary tumour secreting Growth Hormone.
A headache relieved by lying down - Although headaches are common in pituitary tumours, they tend to be made worse by lying down as they are caused by increased intracerebral pressure. A headache relieved by lying down suggests a migraine.
Loss of the outer 3rd of the eyebrows - this is a rare sign of hypothyroidism rather than hyperthyroidism.
Multiple swellings in the midline of the neck - this would suggest a primary cause of hyperthyroidism most likely a toxic multinodular goitre. However, the elevated TSH makes this unlikely, as TSH would be reduced in that case.
A 23-year-old male patient with type one diabetes presents to hospital with symptoms of sweating and dizziness. His capillary blood glucose is currently 1.5 mmol/L and upon examination, you identify fatty lumps around his abdomen. He reports that he has not changed his insulin dose or his diet. He reports having recently started ibuprofen for a sprained ankle.
Which of the following adverse effects is most likely to be causing this patient's symptoms?
E) Lipohypertrophy due to insulin
Repeatedly injecting insulin into the same site leads to the build-up of fatty tissue (lipohypertrophy) at injection sites. Histologically lipohypertrophy is characterised by increased adipocyte size and morphologic variability, accelerated metabolic activity of fat tissue and even adipocyte invasion of the dermis. Repeated needle trauma to a single area is thought to be the primary etiology, however other factors may be involved. Injection of insulin into sites affected by lipohypertrophy can result in sporadic absorption of insulin, potentially leading to either hypoglycemic or hyperglycemic events.
At what level of HbA1c would you consider starting a type 2 diabetic (who has undertaken lifestyle and diet advice) on a first-line pharmacological treatment to better control their blood sugar?
B) > 48mmol/L (6.5%)
Most type 2 diabetics should aim to keep their HbA1c below 48mmol/L.Those in a pre-diabetic state (HbA1c below 48mmol/L) should be given lifestyle and diet advice to try and control their blood sugar. First-line treatment begins when HbA1c increases above 48mmol/L at which point a drug not associated with hypoglycemia should be considered (e.g. Metformin).
Second-line treatment begins when HbA1c increases above 58mmol/L and the first line treatment has been exhausted to the maximum dose with aim of achieving a HbA1c of 53mmol/L. An example of a second-line treatment would be Gliclazide.
Third-line treatment can be administered when HbA1c increases above 58mmol/L.
A 49-year-old man has recently been diagnosed with type 2 diabetes and is being carefully monitored. He has been advised to maintain a healthier diet and lifestyle, he attends a follow-up clinic and claims to have been following the diet stringently since his last appointment three months ago.
Which of the following investigations is most appropriate to assess his glycaemic control since his last review?
E) Glycated hemoglobin
Glycated hemoglobin (HbA1c) is a form of hemoglobin that is measured primarily to identify the three-month average plasma glucose concentration. The test is limited to a three-month average because the lifespan of a red blood cell is four months (120 days). It is formed in a non-enzymatic glycation pathway as a result of hemoglobin's exposure to plasma glucose. Therefore, glycated hemoglobin (HBA1C) is a useful way to assess blood glucose levels over the previous 3 months.
Measuring the random plasma glucose and fasting glucose will show the state of glucose control at the time of measurement, but provides no information about the degree of control the patient has over a longer period of time. Patients may therefore fast closer to the date of their appointment despite poor compliance. Similarly, urine dipstick can only reflect the control of glucose homeostasis at the time of testing. Urine dipstick is also only appropriate as a screening measurement and must be quantified with blood tests. Weight measurement would be useful to record the change in BMI over time; however, it provides no information to the state of diabetes control.
You are a junior doctor on your primary care rotation. You see Pascal, a man recently diagnosed with lung cancer. He wants to discuss a 'rash' that has been gradually arising in his axillae, and also now on his abdomen. While talking to him, you notice that he has moon facies and facial plethora.
To what type of lung cancer can these symptoms all be attributed?
C) Small cell lung cancer
Small cell lung cancer can cause the secretion of:
1. ACTH which leads to Cushing's disease as seen in this example
2. ADH which leads to the syndrome of inappropriate ADH production (SIADH)
3. Antibodies against presynaptic Ca2+ channels leading to Lambert-Eaton Syndrome
Smoking is a significant risk factor for the development of small cell lung cancer.
A 19-year-old university student with a background of type 1 diabetes mellitus presents to ED. He is nauseated and vomiting. He tells you his glucometer has been reading “Hi”. On further questioning, he admits to missing insulin doses occasionally since starting university. Capillary ketones are positive and blood gas reveals a pH of 7.22.
Which is the most appropriate next step in his management?
C) 1L normal saline STAT
This gentleman has presented with diabetic ketoacidosis (DKA). The first step in management involves fluid resuscitation. Most patients are significantly dehydrated due to osmotic diuresis secondary to glycosuria. Insulin infusion is, of course, an important step as is monitoring electrolyte levels (especially potassium), searching for a precipitant (e.g. infection, medication non-compliance etc), education.
Addison's disease results from which of the following processes?
A) Failure of the adrenal glands to produce sufficient aldosterone and cortisol
Addison's disease, which is also known as primary adrenal insufficiency, results from a failure of the adrenal glands to produce sufficient aldosterone and cortisol. This condition is often autoimmune in nature in the developed world and due to tuberculosis in the developing world.
Addison's disease is related to a wide range of clinical features, including: fatigue, abdominal pain, weight loss, skin hyperpigmentation, hypotension, headache, nausea and vomiting, sweating, personality and mood changes, as well as adrenal crisis.
Which of the following thyroid cancers CANNOT be diagnosed on fine needle aspiration cytology alone?
A) Follicular carcinoma
If a patient is suspected to have a thyroid malignancy, they undergo fine needle aspiration cytology to check whether the cells are benign or malignant. This involves putting a fine needle into the thyroid mass and removing a sample of cells with a syringe. This is a very sensitive test to diagnose all the thyroid cancers aside from follicular carcinoma.
Follicular carcinoma cannot be diagnosed in this way as both follicular adenoma and follicular carcinoma have similar cytological appearances. If the fine needle aspiration cytology reveals follicular cells, the patient needs to have a lobectomy and have a formal histological analysis performed to check for evidence of invasion of adjacent structures such as vessels and/or the capsule.
You are an F2 on a COTE hospital ward and at 7PM are called to attend a patient who has collapsed. She is on the ward due to a suspected UTI and has been prescribed insulin due to her diabetes.
Of the following, which is the most appropriate initial investigation?
D) Capillary blood glucose
Capillary blood glucose is a quick investigation to rule in or out the possibility of hypoglycemia given the patient is receiving insulin treatment). The patient has a UTI and potential delirium and it is just past a mealtime. It is therefore possible that she did not eat her full meal but was still given insulin, leading to hypoglycemia and subsequent collapse.
Thelarche is the term for which of the following pubertal changes?
C) Breast budding
Thelarche is the term for breast budding in females, marking the beginning of secondary breast development. Thelarche is usually noticed as a firm, tender lump directly under the centre of the nipple.
An 18-year-old female presents to A&E with a 2-day history of nausea and vomiting. On further questioning, she reports that she has lost around 1 stone in weight over the last 2 weeks. She thinks she has a 'water infection' because she has been going to the toilet a lot. Whilst taking the history, she drinks a full bottle of water. On examination, she has dry mucous membranes, her respiratory rate is 35 and her blood pressure is 126/82. Her abdomen is soft and non-tender. A BM shows a blood glucose of 28.9 mmol/L. A urine dip shows 3+ of ketones. An ABG shows a pH of 7.01.
You commence an insulin infusion and 3 hours later you are reviewing another patient when your crash bleep goes off. The 18-year-old female is in cardiac arrest.
What is the most likely cause of her cardiac arrest?
A) Hypokalemia
The patient in this scenario has diabetic ketoacidosis (DKA) and has therefore been commenced on an insulin infusion. Insulin causes potassium to move into cells which increase the risk of developing hypokalemia. This is the most likely cause of her cardiac arrest. In DKA, potassium should be closely monitored and the patient should be on cardiac monitoring during the acute phase of treatment. Most hospitals provide very detailed DKA management plans, which includes how often to monitor potassium levels and how much replacement to give.
A 19-year-old male is bought in by paramedics after being found unconscious in his room by his flatmate. The paramedics tell you that the male was unresponsive when they arrived and was covered in vomit. Collateral history from his flatmate reveals he has no past medical history and is not currently taking any medications.
You have been asked to put a cannula in as the paramedics struggled to gain IV access and take an ABG.
Observations: O2 sats 98% on air, HR 110, BP 92/65, RR 28, temperature 37°C
ABG results: pH 7.1, PaO2 12.1, PaCO2 2.6, HC03 16
BM: glucose 26mmol/L
A catheter is inserted and the urine tests positive for ketones
Which of the following is the most appropriate immediate management step?
A) 500ml 0.9% Saline over 15 minutes with potassium chloride and a fixed rate insulin infusion
B) 500ml 0.9% Saline over 15 minutes and a fixed rate insulin infusion
C) 500ml 0.9% Saline over 30 minutes with potassium chloride and a rapid acting subcutaneous insulin injection
D) 500ml 0.9% Saline over 30 minutes and a rapid acting subcutaneous insulin injection
B) 500ml 0.9% Saline over 15 minutes and a fixed rate insulin infusion
The history and clinical findings are strongly suggestive of a new diagnosis of diabetic ketoacidosis (DKA).
Diagnosis of DKA requires all 3 of the following to be present:
Blood glucose >11mmol/L
Urine ketones ++
pH <7.3
Acute management includes rapid fluid resuscitation and a fixed rate insulin infusion of 0.1 units / kg/ hour. Once blood glucose falls below 14mmol/L the saline should be replaced with dextro-saline.
Patients can initially present with apparent high potassium, however once the insulin infusion is commenced potassium enters the cells which can result in a life-threatening hypokalaemia, therefore potassium replacement is essential. Potassium replacement dose is dependent of serum levels and therefore since this patient does not have potassium levels available yet, focus should be on fluid resuscitation.
Which investigation would be most beneficial in confirming the underlying cause for the development of an ulcer on the plantar aspect of the foot?
D) Fasting blood glucose
The callus surrounding the ulcer and the location of the ulcer suggests the underlying pathology is peripheral neuropathy, and the most likely aetiology of peripheral neuropathy is diabetes. The presence of peripheral neuropathy results in the loss of pain sensation, preventing the patient from being aware of wounds on their feet. As a result, they continue to walk on the wound generating a callus.
Fasting blood glucose is the best diagnostic test on the list for detecting diabetes.
An ankle-brachial pressure index measurement is helpful for diagnosing arterial insufficiency.
A plain x-ray of the foot would be a useful further investigation for a diabetic ulcer to ensure the patient has not developed osteomyelitis. However, this would not diagnose diabetes.
A biopsy of the wound would show necrotic tissue, but would not directly assist in the diagnosis of diabetes.
A venous duplex scan is helpful for diagnosing venous insufficiency.
A 53-year-old woman has been complaining of tingling and cramping in her hands. She then develops to have a tonic-clonic convulsion. Earlier that day she had a total thyroidectomy for thyroid cancer.
Which of the following is the most likely cause of her seizure?
B) Hypocalcemia
Hypocalcemia is a complication of thyroidectomy due to removal of the parathyroid glands and is the most likely cause of the patient's seizure. The presence of tingling and cramping in the patient's hands is also a typical symptom of hypocalcemia.
Cerebral metastases are unlikely with thyroid cancer.
Sepsis is a complication of surgery but given there is no fever this is unlikely.
Hypoglycemia is possible and should be excluded but unlikely if the patient is not receiving medication lowering her blood glucose level
A 28-year-old female presents to her GP after a positive pregnancy test. Her past medical history is significant for hyperthyroidism, which is currently well controlled on medication. She has no other medical history of note.
Which medication should she take to control her hyperthyroidism at this time?
B) Propylthiouracil
Propylthiouracil is less likely to cross the placenta than carbimazole and methimazole and is usually considered the preferred antithyroid drug in pregnancy. The safest option is often to use propylthiouracil in early pregnancy, changing to carbimazole in the latter months.
Propanolol is used for symptom control as opposed to hormone level management.
Radioactive iodine is contraindicated in pregnancy due to the increased risk of birth defects.
A 53-year-old woman comes to the GP as she has been experiencing hot flushes associated with menopause. She had a thrombotic stroke 3 years ago. There is no other significant medical history. She is a smoker with a 20 pack year history.
Which of the following would be the best treatment for her hot flushes?
D) Sertraline
Hot flushes are a symptom of the menopause. Usually, if they are interfering with someone's life (e.g. leading to poor sleep), Hormone Replacement Therapy (HRT) is usually prescribed (e.g. oestrogen or oestrogen + progesterone).
However, HRT increases the risk of thrombotic diseases such as deep vein thrombosis, pulmonary embolism and stroke, so it is contraindicated in patients who have a history of these conditions. An alternative for patients at increased risk of thrombosis are selective serotonin reuptake inhibitors (SSRIs). Multiple SSRIs have been studied but Sertraline has so far been found to be the most efficacious and best tolerated.
Black Cohosh is a herbal medicine and not prescribed as it is linked to liver problems in rats.
A healthy and asymptomatic 58-year-old lady is found to have the following thyroid function tests in a routine employment healthcare check-up:
TSH: 0.1 mU/L (0.4-4.0 mU/L)
T4: 20 pmol/L (9-24 pmol/L)
T3: 6.1 pmol/L (3.5-7.8 pmol/L)
What is the most appropriate next step in her management?
D) Re-check thyroid function in 3 months
This lady has subclinical hyperthyroidism because her TSH is low but T4/T3 levels are normal and she is asymptomatic. Therefore, the most appropriate to confirm her diagnosis would be to re-check her thyroid function in 3 months. If TSH returns to within the reference range, it is likely that the hypothalamic-pituitary axis was disturbed transiently by a non-thyroidal illness and no further management is necessary.
The other choices are incorrect because of the following reasons:
1. MRI pituitary - This is an appropriate diagnostic test in patients with secondary hypothyroidism of unknown cause but not indicated in subclinical hyperthyroidism, where T3/T4 levels are normal and the patient is asymptomatic.
2. Initiate carbimazole therapy - If the patient's TSH was persistently below 0.1 mU/L, the patient is elderly or has cardiovascular risk factors then this is a possible treatment that may be offered in order to reduce their cardiovascular risk. However, further investigation is important first to confirm the diagnosis.
3. Refer to an endocrinologist - If after 3 months the patient had persistent subclinical hyperthyroidism or became symptomatic, then they should be referred to an endocrinologist for further investigation. However, this step is not necessary at present.
4. Thyroid antibody tests - These are not indicated as the patient's T4/T3 levels are normal.
Which of the following is typically the first sign of puberty onset in females?
C) Breast budding
Breast budding is the first pubertal sign in females, whereas testicular enlargement is the parallel sign in males.
A 32-year-old lady consults her GP because she is worried that her periods have been very heavy and painful recently. She has a past medical history of type 1 diabetes. She also states that her mood has been low recently and she has put on some weight.
Which of the following blood results is most likely to be low?
C) T4
Hypothyroidism is a recognised cause of menorrhagia or abnormally heavy bleeding during menstruation. There is an association between autoimmune hypothyroidism (Hashimoto's thyroiditis) and other autoimmune diseases, such as this lady's Diabetes Type 1. She also has signs suggesting hypothyroidism such as the low mood and weight gain. The correct answer is, therefore, T4, which would be low in hypothyroidism.
The other choices are incorrect because of the following reasons:
1. TSH - This lady has a history of autoimmune disease (type 1 diabetes) and therefore most likely has primary hypothyroidism caused by Hashimoto's thyroiditis as they are strongly associated. In primary hypothyroidism, TSH is high due to the loss of negative feedback of thyroid hormones on the pituitary.
2. Testosterone - The differential diagnosis being tested here is polycystic ovarian syndrome (PCOS), which would lead to irregular and heavy periods. However, if PCOS was the correct diagnosis, testosterone would be increased rather than decreased.
3. Total iron binding capacity (TIBC) - Menorrhagia is a common cause of iron deficiency in women. It would be associated with a raised TIBC due to increased production of transferrin by the liver in order to maximise the use of the little iron that is available.
4. Folate - Folate deficiency tends to be caused by reduced dietary intake or malabsorption, but not bleeding. Folate deficiency can cause non-specific symptoms such as fatigue, diarrhea and glossitis, and is a particular worry in women of childbearing age because it can cause neural tube defects in the developing fetus. Pernicious anemia, which causes Vitamin B12 deficiency, is associated with type 1 diabetes.
A 21-year-old female presents with decreased level of consciousness. She was discovered unresponsive in a shop entryway by a member of the public, who called an ambulance. Her medical history is unknown. She is unable to give a history. Results of the primary survey and initial investigations include:
Primary Survey:
Airway: Maintained, mumbling and incoherent, sweet-smelling breath
Breathing: Chest clear, Sa02 = 100% on Fi02 0.21, respiratory rate = 25
Circulation: HS 1+2, BP = 106/75 mmHg, heart rate = 112, peripherally cool, capillary refill time less than two seconds, dry mucous membranes
Disability: GCS = 11, blood glucose = 16 mmol/L, temperature = 34.5 °C
Exposure: Abdomen mildly tender, no external signs of injury.
Investigations:
ECG: sinus tachycardia with a heart rate of 115
Arterial blood gas:
pH
7.2 kPa ( 7.35-7.45)
Pa02
14 kPa (11-13kPa)
PaC02
3.8 kPa (4.7-6.0)
HCO3
16 mEq/L (22-26)
Which single investigation is most appropriate to reveal the underlying diagnosis?
E) Serum ketones
This patient’s presentation with decreased level of consciousness with associated hyperglycemia and acidosis is most likely due to diabetic ketoacidosis (DKA). This may be the first time the patient has presented with symptoms of type 1 diabetes mellitus, or the presentation may be due to a lack of adherence to regular insulin therapy. Due to the build up of ketones in the body, serum ketones are the most appropriate investigation to confirm the diagnosis.
Decreased level of consciousness is a sign of severe DKA and would prompt consideration of referral to a high dependency area. Immediate treatment of diabetic ketoacidosis aims to correct the metabolic abnormality with fluid replacement, monitoring and replacement of serum potassium and glucose. It is also vital to consider and treat any precipitating cause for this presentation, such as infection.
CT head can diagnose structural abnormalities within the brain, such as a space-occupying lesion and cerebral hemorrhage. However, there are no focal neurological signs or clear history of head injury in this case. Cerebral oedema is a rare and serious complication of diabetic ketoacidosis and is associated with use of large volume fluid resuscitation. Cerebral imaging would be indicated if this patient’s consciousness level fails to improve or worsens after initiation of treatment.
Excess alcohol can impair consciousness and cause ketoacidosis. Alcoholic ketoacidosis is associated with low or normal serum glucose levels. However, serum alcohol levels should not be tested before serum ketones due to the low diagnostic power.
Urine drug screening may identify contributory causes to the patient’s decreased level of consciousness but would not account for the metabolic abnormalities present.
Chest X-ray would be indicated if there are any concerns about the possibility of infection as a precipitating cause but would not confirm a diagnosis of diabetic ketoacidosis.
What is the target blood pressure in adults aged less than 80 with type 2 diabetes mellitus and no other comorbidities?
D) 140/90
NICE recommends the following blood pressure targets for patients with type 2 diabetes and no other co-morbidities:
1. Aged under 80 — reduce clinic blood pressure to below 140/90 mmHg and maintain that level. Use clinical judgement for people with frailty or multi-morbidities.
2. Aged 80 and over — reduce clinic blood pressure to below 150/90 mmHg and maintain that level. Use clinical judgement for people with frailty or multi-morbidities.
Previously, NICE suggested a target blood pressure of 130/80 mmHg for patients with type 2 diabetes mellitus in the presence of target organ damage (such as eye, kidney or cerebrovascular disease). However, recent studies have demonstrated that there is insufficient evidence to aim for lower blood pressure in this age group, and so the target blood pressure is now 140/90 for all patients aged under 80.
A 24-year-old woman presents to you with amenorrhoea. Her last menstrual period was 4 months ago and are usually irregular. She tells you that she has been suffering from depression since the age of 16 because she has acne and a 'hairy face'. She is also finding it difficult to lose weight. Her BMI is 30 kg/m2. She is currently on sertraline and benzoyl peroxide. You perform a pregnancy test which is negative.
What is the most likely diagnosis?
A) Polycystic ovary syndrome
PCOS is an endocrine disorder with clinical features such as hirsutism, acne, oligomenorrhoea/amenorrhoea and multiple cysts in the ovary. The cause is unknown.
PCOS is diagnosed if ≥2 of the following criteria are present:
1. Oligomenorrhoea/amenorrhoea
2. Clinical or biochemical signs of hyperandrogenism (e.g. hirsutism, acne)
3. Polycystic ovaries on ultrasound scan (12 or more follicles in at least one ovary).
Management depends on whether or not the woman wishes to conceive. If she is trying to become pregnant, clomifene is prescribed to induce ovulation. Metformin may be used as an alternative or in addition to this. If fertility is not desired, the combined oral contraceptive pill may be used to regulate the menstrual cycle and improve symptoms of acne and hirsutism. The most common complication of PCOS is infertility (75%).
Which one of the following diabetes medication classes has been shown to be weight-neutral/weight-losing in patients with T2DM?
A) SGLT-2 inhibitors
SGLT-2 inhibitors inhibit the sodium-glucose transporter-2 protein that facilitates the resorption of sodium and glucose from the proximal convoluted tubule to the glomerular capillaries. This leads to increased excretion of glucose in the urine (glycosuria). This effect has been shown to aid patients with T2DM lose weight. The other medications listed can cause weight gain.
Weight status is important when selecting oral diabetes medications for T2DM. Obesity is associated with T2DM and indeed weight loss in overweight/obese patients can improve glycaemic control in T2DM patients. Practitioners have scope to choose from several diabetes medications when adding onto metformin monotherapy. Obesity is one factor to consider when prescribing diabetes medications. Think about patient factors (eg. presence of co-morbidities) and medication factors (eg. cost, efficacy, side effects) when prescribing medications for patients, particularly when guidelines offer clinicians scope of choice.
Ms. Jones is a 30-year-old female who presents with a 3-month history of 10kg weight loss despite an increased appetite, anxiety, diarrhoea and amenorrhoea.
Examination reveals a fine tremor, brisk reflexes and a systolic murmur heard throughout the precordium. A urine pregnancy screen was negative.
What is the most likely finding on examining the pulse?
A) Irregularly irregular pulse
The patient's symptoms and examination findings are consistent with a diagnosis of hyperthyroidism. The systolic murmur is likely to be a functional murmur due to a hyperdynamic circulation caused by the hyperthyroidism.
Hyperthyroidism can cause atrial fibrillation which would cause an irregularly irregular pulse. The pulse may also be fast and regular (sinus tachycardia) or have a normal rate and rhythm; however, these options were not available.
The other choices are incorrect because:
1. A low volume pulse is associated with aortic stenosis and hypovolemia.
2. A slow rising pulse is associated with aortic stenosis.
3. A collapsing pulse (sudden, abrupt drop in pulse pressure after ventricular systole due to the regurgitation of blood back into the left ventricle) is associated with aortic regurgitation.
4. A bounding pulse is associated with sepsis and hypercapnia.
5. Pulsus paradoxus (a large decrease in pulse pressure during inspiration) is a sign of various conditions, including cardiac tamponade, constrictive pericarditis, pulmonary embolism, and acute asthma.
6. Pulsus alternans (significant beat to beat variation in the pulse pressure) is associated with severe ventricular failure.
A 74-year-old gentleman is admitted with new onset confusion and a fever of 39.5 degrees. On admission, the junior doctor takes blood tests to rule in/out various causes of confusion.
Thyroid function tests (TFTs) show the following:
TSH: 0.1 mU/L (0.4-4.0 mU/L)
T4: 6 pmol/L (9-24 pmol/L)
T3: 2.9 pmol/L (3.5-7.8 pmol/L)
What is the most likely cause of this patient's abnormal TFTs?
E) Euthyroid sick syndrome
This patient has both a low TSH and a low T4. Therefore, the most likely cause of this patient's abnormal thyroid function tests is euthyroid sick syndrome. Euthyroid sick syndrome is a condition in which serum levels of thyroid hormones are low in clinically euthyroid patients with non-thyroidal systemic illness. Diagnosis is based on excluding hypothyroidism via the measurement of TSH (high in hypothyroidism and normal/low in euthyroid sick syndrome). Treatment is directed toward the underlying illness; thyroid hormone replacement is not indicated.
Subclinical hypothyroidism is found incidentally on blood testing as the patient typically has no symptoms or clinical signs. Blood tests show a raised TSH and a normal T4/T3.
Hashimoto's thyroiditis is the most common cause of hypothyroidism. It is an autoimmune disease in which the thyroid gland is gradually destroyed. It presents with features of hypothyroidism (e.g. weight gain, fatigue, cold intolerance) and sometimes with a painless goitre. Blood tests show a raised TSH and low T4/T3. Diagnosis is confirmed by the presence of anti-thyroid peroxidase antibodies.
De Quervain's thyroiditis typically presents after an upper respiratory tract infection, initially with features of hyperthyroidism and a tender goitre (as the thyroid hormones spill into the circulation from the inflamed thyroid tissue). Once the acute inflammation is over, the patient can then become hypothyroid.
Secondary hypothyroidism (such as a pituitary tumour compression TSH secreting cells) typically presents with a low TSH and low T3/T4 as in this clinical scenario. The absence of any headache or visual symptoms (e.g. optic chiasmal compression) and the presence of an acute infective illness makes euthyroid sick syndrome a more likely diagnosis.
Which one of the following visual field defects are classically associated with a pituitary adenoma?
D) Bitemporal hemianopia
Bitemporal hemianopia involves the loss of vision in the outer half of both the right and left visual field (sometimes described as tunnel vision). It is usually associated with lesions of the optic chiasm, the area where the optic nerves from the right and left eyes cross near the pituitary gland. Bitemporal hemianopia most commonly occurs as a result of tumours located at the optic chiasm. Since the adjacent structure is the pituitary gland, some common tumours causing compression are pituitary adenomas and craniopharyngiomas. Aneurysms of the anterior communicating arteries which arise superior to the chiasm can enlarge and compress the chiasm from above.
Homonymous hemianopia involves visual field loss on the left or right side of the vertical midline, usually affecting both eyes. Vascular (e.g. stroke) and neoplastic lesions (e.g. benign/malignant tumours) from the optic tract, to the visual cortex, can cause a contralateral homonymous hemianopsia.
Central scotoma involves an area of decreased vision that corresponds to the point of fixation and interferes with central vision. It suggests a lesion between the optic nerve head and the chiasm (e.g. multiple sclerosis/glaucoma).
Inferior homonymous quadrantanopia involves the loss of a quarter of the inferior visual field in both eyes. A parietal lesion (e.g. stroke) causing interruption of the superior optic radiations can result in an inferior homonymous quadrantanopia on the contralateral side of both eyes.
A 49-year-old woman is investigated following an osteoporotic hip fracture. The following results are obtained:
Which one of the following autoantibodies is most likely to be present?
C) TSH receptor stimulating autoantibodies
This patient most likely has Graves' disease, a common cause of thyrotoxicosis.
In this condition, there is an overproduction of T3 and T4. The T3 increases osteoclastic activity (bone resorption), hence this lady has osteoporosis which would normally be uncommon in a person of her age.
Epidemiology of the disease statistically shows it is most typically seen in women aged 30-50 years.
Other autoantibodies suggestive of autoimmune thyroid disease include anti-thyroid-peroxidase (anti-TPO) antibodies and anti-thyroglobulin(anti-Tg) antibodies. These are non-specific and can be present in autoimmune causes of both hyperthyroidism (Grave’s disease) and hypothyroidism (Hashimoto’s disease).
A 76-year-old gentleman with a history of primary hypothyroidism attends his GP for the first time in many years for a routine check-up. He divulges that he has not been complying fully with his levothyroxine therapy as he sometimes cannot make it to a pharmacy to refill his prescription.
Which of the values below is most likely to be low in this gentleman?
E) Core temperature
Hypothyroidism reduces the basal metabolic rate, leading to cold intolerance and reduced core temperature as less heat is produced as a by-product of metabolism.
Mean cell volume: Macrocytosis is found in up to 60% of patients with hypothyroidism and can result from deficiencies in either thyroid hormone itself, or in vitamin B12 (Pernicious anaemia occurs 20 times more frequently in patients with hypothyroidism). The other main type of anaemia seen in hypothyroidism is normocytic anaemia, as thyroid hormone stimulates the production of erythropoietin (EPO) in the kidney. Therefore, it is unlikely that this gentleman with poorly managed hypothyroidism would have a low MCV, but rather it would be high or normal.
LDL Cholesterol: One of the major reasons for treating hypothyroidism is that it leads to a significant rise in serum LDL Cholesterol, which causes atherosclerosis and premature heart disease. The rise in LDL can be explained by a reduction in circulating T3, which at normal concentrations up-regulates the LDL-receptor, allowing LDL cholesterol to be catabolised.
TSH: In primary hypothyroidism, TSH levels should increase due to the loss of negative feedback of thyroid hormones on the pituitary. Therefore, TSH would likely be high in this man who has poorly managed primary hypothyroidism.
Body mass index: Hypothyroidism causes an increase in BMI due to a reduction in basal metabolic rate, which decreases calorific expenditure and subsequently causes weight gain. Therefore, BMI would likely be high in this man who has poorly managed hypothyroidism.
A 25-year-old female presents with profuse sweating, palpitations, heat intolerance, menstrual abnormalities, and weight loss. She is subsequently diagnosed with hyperthyroidism and prescribed propylthiouracil 300 mg. She returns for follow-up after 2 weeks and is found to have a fever of 38.5° C, sore throat, and swollen tonsils. What is the best next step in her management?
B) Stop propylthiouracil
The best next step in management is to stop propylthiouracil. Antithyroid medications including propylthiouracil and methimazole have several side effects, the most dangerous being agranulocytosis. Due this adverse effect, any patient presenting with sore throat or fever must be immediately taken off the antithyroid medications, and their white blood cell count should be measured to confirm the diagnosis. A cell count of <500/μl in a patient prescribed antithyroid medications is diagnostic.
Beta-blockers can be used to treat thyrotoxic symptoms, but sore throat does not warrant beta-blocker use. Increasing the dosage will worsen the patient’s condition. Starting paracetamol and throat culture with antibiotics could be considered but would not be the best next step.
A 29-year-old male is being evaluated for bilateral breast enlargement. Despite frequent unprotected sexual intercourse with his female partner, the patient states that they have not been able to conceive for the last 18 months. On physical examination, he has a tall stature and his testicles are small and firm.
Which of the following would you expect to see on laboratory evaluation?
A) Increased plasma follicle-stimulating hormone
In the above scenario, the patient presents to the clinic with the classical signs and symptoms of Klinefelter's syndrome. Klinefelter's syndrome accounts for approximately 1/1000 live male births. This syndrome is the clinical manifestation of a male who has an extra X chromosome with the most common genotype being 47, XXY. These patients classically present with a distinctively elongated body habitus and small atrophic testes on physical examination. Affected individuals also have a lack of secondary sexual characteristics like a deep voice, facial hair and pubic hair. Due to dysgenesis of the seminiferous tubules, there is a reduction in inhibin B seen in these patients. This leads to a lack of negative feedback on the anterior pituitary and consequently an elevated FSH on laboratory testing.
Variations in TSH are not typically seen in patients who have Klinefelter's syndrome. TSH levels are typically low in primary hyperthyroidism and secondary hypothyroidism.
Estradiol is elevated in patients with Klinefelter's syndrome as opposed to being low.
Prolactin levels can be increased in prolactinomas, usage of certain medications (antipsychotics, metoclopramide, ranitidine etc) but are unlikely to be elevated in a patient with Klinefelter's syndrome.
Dysgenesis of the seminiferous tubules in Klinefelter's syndrome leads to low inhibin B levels.
Jenny Cooper is an 18-year-old girl. She is referred to a gynaecologist with primary amenorrhoea. She is slim and sporty. She exercises 5 times a week and has a good diet. Previous medical history is unremarkable.
On examination, she has:
BMI 18
Tanner stage 4-5 breast development
An absence of body hair
Normal external genitalia
Palpable masses in the groin
She is sent for karyotyping, hormone level testing and a transabdominal ultrasound scan.
From the information above, what is her most likely diagnosis?
D) Complete androgen insensitivity syndrome
Complete Androgen Insensitivity Syndrome is the most likely diagnosis. It would be confirmed by karyotyping showing XY and very high Testosterone levels. The condition is due to testosterone being completely inactive so male internal genitalia or external genitalia do not develop. Sertoli cells are still present and produce anti-Mullerian hormone, so the Mullerian duct regresses meaning female internal genitalia are not produced. The lack of DHT (made from testosterone) causes female external genitalia to develop.
Partial Androgen Insensitivity Syndrome and Congenital adrenal hyperplasia would lead to ambiguous external genitalia, however, this is not the case with Jenny as she has 'normal genitalia'.
Turner's syndrome (XO genotype) could be a possible diagnosis but usually effects growth and development which are not impaired in Jenny.
A 34-year-old woman presents with polyuria and polydipsia. This has been ongoing for the past 3 months. She has also been constipated over this period. Incidentally, she mentions that she has been having intermittent episodes of flushing and headaches that have been ongoing for the past 1 month.
Blood tests have shown the following:
Urea: 5.0
Creatinine: 80
Sodium: 137
Potassium: 4.2
Calcium: 3.1
The examination was unremarkable and the patient appears of normal height and no skin/mucosal changes were noted. A lump was palpable on her neck. Her observations also show that she has a blood pressure of 190/120mmHg. She has been told her blood pressure was high at her last clinic appointment. She also noted that her mother and sister have had thyroid surgery in the past but could not remember why.
What is the most likely underlying diagnosis?
C) Multiple endocrine neoplasia 2a (MEN-2a)
he constellation of symptoms of hypercalcemia, hypertension, and the family history of thyroid surgery makes MEN-2a the most likely diagnosis in this case. MEN-2a typically presents with medullary thyroid cancer (MTC), parathyroid hyperplasia, and pheochromocytoma. This is inherited in an autosomal dominant pattern, explaining the family history that is presented.
MEN-1 will typically present with a pituitary adenoma. It is also possible for the patient to develop hyperparathyroidism and pancreatic tumours, such as insulinoma.
MEN-2b is associated with both parathyroid hyperplasia and pheochromocytoma. However, the absence of a marfanoid body appearance makes this less likely. MEN-2b will typically show signs of mucocutaneous neuromas.
NF-1 is caused by a mutation of chromosome 17 affecting the ras-pathway. This is not associated with either MTC or pheochromocytoma. This presents with skin lesions such as café au lait spots, axillary freckling, dermal neurofibromas, and Lisch nodules.
VHL does not typically involve parathyroid hyperplasia. It can present with pheochromocytoma but is also more commonly seen with hemangioblastomas (both retinal and in the central nervous system), pancreatic cysts, and renal cell carcinoma.
Miss Greenhill is on your surgical ward following her parathyroidectomy for primary hyperparathyroidism. You have been called to assess her because she is becoming increasingly short of breath.
The nurse gives you her observations which are: RR 24, SpO2 91% on air, HR 118, BP 108/64, Temp 37.9. He also tells you that Miss Greenhill did not have anything to eat for dinner as she was having difficulties swallowing.
On examination Miss Greenhill is in obvious distress, she answers questions one word at a time with increased respiratory effort, she has no added respiratory or cardiac sounds and her pupils are equal and reactive. Her abdomen and calves are soft and non-tender, she has no obvious rashes or skin changes except for bruising and swelling around her surgical site.
You sit Miss Greenhill up in her bed and apply 15L oxygen via a non-rebreathe mask. As you speak to her she begins to become drowsy and unresponsive and her oxygen saturations drop to 83%. You call the surgical registrar and CRASH team who are both en-route to the ward.
What is the most appropriate step to definitively manage Miss Greenhill's underlying problem?
E) Open her surgical wound
This patient has a neck hematoma, a rare but potentially fatal complication following surgery to the neck. It occurs due to post-surgical bleeding within the neck, forming an expanding hematoma which, left untreated, will lead to compression of the trachea eventually causing complete airway obstruction.
In a peri-arrest situation, immediate (on the ward) evacuation of the clot is required to stop a cardiac arrest. This is done by opening three surgical layers; first the superficial skin (which may be sutured, glued or stapled), second the platysma muscle layer (sutured) and third is the strap muscle of the neck (sutures). For this reason, all post-neck surgery patient should have a "hematoma evacuation box" sent to their post-op ward with them. This will include a scalpel +/- scissors +/- staple remover with gauze/dressings.
The other choices are incorrect because:
1. Commencing CPR will be the correct response should the patient go into cardiac arrest.
2. IM adrenaline is an appropriate response in anaphylaxis.
3. Cannula into 2nd ICS MCL is classically the initial management of a tension pneumothorax.
4. Treatment dose LMWH would be appropriate for pulmonary embolism. LMWH is very inappropriate in neck hematoma as it may worsen bleeding. Neck surgery patients are some of the few instances where VTE post-surgery prophylaxis is not recommended.
What condition results from the over-secretion of growth hormone BEFORE adolescence?
B) Gigantism
Gigantism results from the over-secretion of growth hormone prior to the onset of adolescence, specifically prior to the fusion of epiphyses. Excessive growth hormone production associated with gigantism is most commonly due to the presence of a pituitary tumour. This condition presents with features resulting from excessive growth, including: increased height (sometimes between 7 and 9ft), spade-like hands and feet, prominent forehead and jaw, coarsening of facial features, visual disturbances, as well as other features.
Acromegaly is most commonly mistaken for gigantism. This condition results from excessive secretion of growth hormone after adolescence.
A 53-year-old male presents to their GP complaining of 'feeling unbearably hot' for the last few weeks. On further questioning, the patient reveals this has been associated with his bowels being more 'irregular and loose' lately, with a ravenous appetite. There is no history of recent travel, night sweats or cough. There has been some recent weight loss of around 4kg. He currently lives with his wife and 2 children. He is currently taking Amlodipine for mild hypertension.
On examination, the patient is alert and orientated. His hands have a notable fine tremor and there is some detachment of the fingernails from the nail bed. There is also a swelling below the laryngeal prominence. Observations are normal apart from an irregularly irregular tachycardia at 112 bpm.
What is the most likely diagnosis given his current presentation?
E) Grave's disease
Heat intolerance (due to the increase in non-shivering thermogenesis), onycholysis (detachment of nails from the nail bed), diarrhoea, weight loss, increase in appetite along with general agitation/anxiety are hallmarks of hyperthyroidism. Even though there is typically weight loss in overactive thyroid conditions, 10% paradoxically gain weight due to the increased appetite. Therefore Grave's disease is the most likely diagnosis at this early stage.
Euthyroid hyperthyroxinemia is a condition that often presents with no signs/symptoms of thyroid dysfunction, and is confirmed by a serum blood test that shows normal TSH with high T3/T4. Given that the question has no mention of any serological tests, and the patient is symptomatic, it is unlikely to be this.
Hypothyroidism (e.g. Hashimoto's thyroiditis) is likely to present with lethargy rather than agitation, and cold intolerance rather than feeling hot. (i.e. many layers of clothing on a warm day). Weight gain is a feature of hypothyroidism, but given the other presenting symptoms with this gentleman, it is extremely unlikely to be an under activity of the thyroid gland.
Neuroleptic malignant syndrome (NMS) is a complication (often life-threatening) of taking antipsychotic medications such as haloperidol. The heat intolerance could be suggestive of fever, which is a major feature of NMS along with tachycardia, however, you would expect a history of antipsychotic medication, along with muscle cramps and perhaps seizures.
A pheochromocytoma is an adrenaline producing tumour of the chromaffin cells in the adrenal medulla. It classically presents with a triad of headaches, palpitations and sweating. Although the sweating could be associated with the 'fever' and the palpitations with the irregular heartbeat, the patient has not come in complaining of those specific and slightly distinct ailments. Also the other symptoms in combination paint a more hyperthyroid picture.
A 22-year-old man presents to the emergency department with confusion, abdominal pain, nausea, and vomiting. He is a known type 1 diabetic and is subsequently diagnosed with diabetic ketoacidosis (DKA). His medications include insulin aspart (short-acting insulin), insulin glargine (long-acting insulin), loratadine, ibuprofen and levothyroxine.
Which of the following medications should be withheld while he receives treatment?
C) Insulin aspart
During DKA, patients are given an aggressive fluid replacement and commenced on a fixed rate of insulin infusion. While on the insulin infusion, long-acting insulin should continue, but short-acting insulins should be stopped. Once the patient is biochemically stable and able to eat, the short-acting insulin can restart. The fixed-rate insulin infusion can stop at least 30 minutes after the short-acting insulin has been given.
Ibuprofen, loratadine, and levothyroxine are all safe to continue during treatment for DKA, and stopping them could cause the patient's other medical conditions to become less well-controlled, potentially worsening their diabetic control.
A 30-year-old lady presents to A&E with shortness of breath and vomiting. She reports a productive cough with green sputum, with progressive shortness of breath since the beginning of last week. She was prescribed antibiotics and steroid tablets by her GP a few days ago, with her respiratory symptoms having improved since treatment started. The vomiting started about 12 hours ago and she has felt progressively more fatigued and unwell since then. She also reports passing large volumes of urine over the last 24 hours. She has a past medical history of asthma and type 1 diabetes. On examination, she is clinically hypovolaemic, with dry mucous membranes and a prolonged capillary refill time. Chest examination reveals some left basal crackles and some expiratory wheeze, but good overall air entry. There is some non-specific abdominal tenderness on palpation, but no evidence of peritonism or organomegaly. Chest X-ray does not show any focal consolidation. An ECG appears to demonstrate tall tented T waves.
Vital signs reveal the following:
BP: 100/70
Pulse: 105 bpm
Sats: 98% on air
Respiratory rate: 20
Temperature: 36.3
What is the most likely underlying diagnosis?
C) Diabetic ketoacidosis
The most likely underlying diagnosis is diabetic ketoacidosis (DKA). The history suggests the patient has initially developed an infective exacerbation of asthma, which has been treated with oral antibiotics and steroids. Despite the respiratory symptoms improving, there appears to have been a sudden deterioration in the patient's overall health in the last 24 hours. Some key symptoms mentioned here include vomiting and polydipsia. On examination, the patient appears to be hypovolemic, but the respiratory system appears to be stable. This overall clinical picture would fit with the development of diabetic ketoacidosis that has been triggered by an acute respiratory illness and the consumption of corticosteroids. The extra physiological stress of acute illness can increase insulin demands and corticosteroids also increase blood glucose levels, further increasing insulin demand. The presence of polyuria and vomiting are both highly suggestive of DKA and tall tented T waves on the ECG again would fit with this diagnosis (hyperkalemia secondary to DKA).
Norovirus presents with sudden onset diarrhea, vomiting and abdominal pain. Although the patient has vomiting, the presence of polyuria and the background of type 1 diabetes makes diabetic ketoacidosis a more likely diagnosis.
Panic attacks typically present with hyperventilation, peri-oral tingling (or other transient paresthesias) and anxiety. The patient's respiratory rate is not excessively high, there is no mention of anxiety and no mention of paresthesia.
Addisonian crisis is something not to be missed as it can rapidly lead to death. In this case, it would seem unlikely given the patient has no history of Addison's disease and is on corticosteroid therapy for her chest, meaning she would not be completely depleted of corticosteroid, even if she had stopped producing it. Other symptoms of Addison's disease include hypotension and general fatigue. Patients who are known to have Addison's disease need to double their corticosteroid dose during acute illness to meet the bodies increased demands under physiological stress.
Pulmonary emboli typically present with pleuritic chest pain, sudden onset shortness of breath and hypoxia. There is no mention of chest pain and the shortness of breath came on gradually, with associated green sputum making a lower respiratory tract infection a more likely cause of the shortness of breath. In addition, the patient's oxygen levels are adequate, making the diagnosis unlikely.
A 61-year-old lady who commenced levothyroxine treatment for primary hypothyroidism 2 weeks ago attends a follow-up appointment with her GP, who is concerned because her thyroid function tests do not seem to have normalised. Results from her test taken yesterday are as follows:
TSH: 6 mU/L
T4: 12 pmol/L
What is the most likely explanation for the lady's abnormal thyroid function tests?
E) Treated hypothyroidism
This lady is showing an appropriate response to levothyroxine therapy 2 weeks after initiating it. Thyroid function tests should be only be taken every 6-8 weeks because this is how long it takes for a change in TSH to become evident. Once TSH levels are stable, thyroid function tests should then be repeated annually or earlier if there is clinical suspicion of over- or under-treatment.
Euthyroid sick syndrome- Euthyroid sick syndrome is a condition in which serum levels of thyroid hormones are low in clinically euthyroid patients with a systemic illness that is not caused by a thyroid problem. Diagnosis is based on excluding hypothyroidism via the measurement of TSH (high in hypothyroidism and normal/low in euthyroid sick syndrome).
Subclinical hypothyroidism- is found incidentally on blood testing as the patient typically has no symptoms or clinical signs. Blood tests show a raised TSH and a normal T4/T3, as in this case. However, the result must be interpreted in the context of the patient recently beginning levothyroxine therapy, therefore, this is not the 'best' answer in this case.
TSH secreting pituitary adenoma- This would present with both a raised TSH and T4, and is often associated with other symptoms such as headaches and bitemporal hemianopia, as well as clinical features of hyperthyroidism, not hypothyroidism.
De Quervain's thyroiditis- typically presents after an upper respiratory tract infection, initially with features of hyperthyroidism and a tender goitre (as the thyroid hormones spill into the circulation from the inflamed thyroid tissue). Once the acute inflammation is over, the patient can then become hypothyroid.
A 65-year-old woman presents to her GP complaining of low mood for the past 3 months. She has also noticed she has gone up 2 dress sizes over a similar time period. She reports a past medical history of atrial fibrillation and type 2 diabetes. Clinical examination reveals cool peripheries and a pulse of 45 bpm.
Thyroid function tests (TFTs) show the following:
TSH: 6.5 mU/L (0.4-4.0 mU/L)
T4: 4 pmol/L (9-24 pmol/L)
Which of the following medications is the most likely cause of this woman's symptoms?
C) Amiodarone
This lady has drug-induced hypothyroidism.
Amiodarone is an antiarrhythmic medication used to treat and prevent a number of types of irregular heartbeats (including atrial fibrillation which this lady has). It is known to cause thyroid problems, including both hyperthyroidism and hypothyroidism. Amiodarone is structurally similar to thyroxine and also contains iodine. Both of these contribute to the effects of amiodarone on thyroid function. Amiodarone also causes an anti-thyroid action, via Wolff–Chaikoff effect, due to its large amount of iodine in its molecule, which causes a particular "cardiac hypothyroidism " with bradycardia and arrhythmia.
Lithium can cause hypothyroidism, however, it is typically used in the treatment of Bipolar Disorder, which this patient has no history of.
Metformin is not known to cause hypothyroidism and also is known to encourage weight loss (rather than weight gain in this scenario)
Gliclazide is a sulphonylurea medication used to treat type 2 diabetes. Although it can cause weight gain, it is not known to cause hypothyroidism and therefore would not explain the TFT results.
Carbimazole is a thyroid peroxidase inhibitor that is sometimes used to treat hyperthyroidism in Graves' disease (e.g. block and replace therapy). This drug can cause hypothyroidism, however, there is no mention of a previous history of hyperthyroidism, meaning the patient is highly unlikely to be taking this medication.
Necrobiosis lipoidica is associated with which of the following conditions?
A) Diabetes mellitus
Necrobiosis lipoidica presents as an orange-brown colouration of the shins that progressively becomes atrophic and shiny with telangiectasia. This skin is vulnerable to ulceration. Necrobiosis lipoidica is associated with diabetes mellitus and rheumatoid arthritis.
Sickle cell anaemia can cause vasculitic lesions which have a punched-out appearance with an inflammatory, purpuric edge.
Venous insufficiency causes shallow ulcers that are typically located over the medial malleolus.
Calciphylaxis causes livedo reticularis - a net-like mottled pattern which can ulcerate and become necrotic.
Peripheral arterial disease causes deep punched-out painful ulcers at the distal extremities.
Which of the following would you expect from an individual with a 46XY genotype and deletion of the SRY gene?
E) Female phenotype
Some gene mutations block the action of SRY in development. Thus, a 46XY individual with SRY deletion or mutation would develop with a female phenotype, which is known as Swyer syndrome. Conversely, a 46XX individual with translocation of SRY to an X chromosome would develop with a male phenotype. Individuals with Swyer syndrome do not have normal fertility; they do not have functional ovaries and require hormone replacement therapy later in life in order to develop secondary sex characteristics.
Although people with Swyer syndrome do not develop testes and therefore are not predisposed to this type of cancer, the residual gonadal tissue that would have developed into testes can become cancerous and should be removed.
Mental handicaps are more likely to be observed in other disorders of sex development, such as Turner's syndrome.
A 47-year-old man with type 2 diabetes mellitus presents to his GP for review. He has been taking metformin and pioglitazone for the last 6 months. His most recent HbA1c is 62 mmol/mol. He is not currently keen to start insulin. Therefore he is started on canagliflozin as part of triple therapy.
Which of the following does this new medication increase his risk of?
D) Skin ulcers
Patients with type 2 diabetes taking two antidiabetic medications should have a target HbA1c of <48 mmol/mol . As this patient is still above this, they need to be prescribed either insulin or a third antidiabetic medication. In this case, the regime chosen by the GP is metformin, pioglitazone, and canagliflozin. Canagliflozin is a sodium-glucose co-transport 2 inhibitor (SGLT-2 inhibitor). Patients taking these need close monitoring of their legs and feet as they have an increased risk of ulcers and infections, which can lead to amputation.
Patients taking pioglitazone have a slightly increased risk of bladder cancer - but as long as they respond adequately, the benefits are still thought to outweigh this risk. However, this is not a known risk associated with canagliflozin, which is the medication this question is asking about.
The risk of heart failure is increased in patients who are taking both pioglitazone and insulin together. It is not known to be associated with canagliflozin.
Patients with reduced renal function are at an increased risk of lactic acidosis if they are taking metformin. It is not known to be associated with canagliflozin.
Weight gain is associated with pioglitazone, which this patient is also taking. Canagliflozin often has the opposite effect, and many patients lose weight while taking it.
You are reviewing a patient with moon face, buffalo hump, abdominal striae, and lethargy. A 24-hour urine test to look for free cortisol reveals an abnormal result. This leads you to the diagnosis of Cushing syndrome. You then order a serum ACTH to confirm the cause of the syndrome, which shows raised ACTH. You then give 8mg of dexamethasone as part of a cortisol suppression test and find that cortisol is not suppressed.
Given these results, which of the following is the most likely cause of this patient's condition?
E) Ectopic ACTH tumor
24-hour urine free cortisol may be used to assess cortisol production over the course of the day to account for diurnal variation in cortisol. Measuring serum ACTH will then enable you to elicit whether the tumour is producing ACTH or cortisol. A cortisol producing adrenal tumour will lower ACTH. Hence adrenal gland tumour is incorrect.
A dexamethasone suppression test will suppress endogenous ACTH production in the anterior pituitary gland, leading to cortisol suppression. Hence anterior pituitary tumour and Cushing's disease are incorrect.
Whereas in an ectopic tumour, cortisol is not suppressed, as seen in this case.
Obesity is a sign of Cushing syndrome, not a cause
A 31-year-old woman presents to GP complaining of lethargy, weight loss, and anorexia. On examination, you note patches of depigmented skin on her upper limbs and increased pigmentation in her palmar creases.
Which of the following is the most likely cause of her symptoms?
E) Addison's disease
The symptoms of lethargy, weight loss, and anorexia are suggestive of adrenal insufficiency. This, in combination with vitiligo (patches of depigmented skin) and hyperpigmentation of the palmar creases, would be in keeping with Addison's disease.
Cushing's syndrome is a result of excess cortisol production. It would present with weight gain, a rounded face, easy bruising and weakness.
Depression could present with symptoms of lethargy, weight loss, and anorexia. However, the vitiligo and hyperpigmented palmar creases would not be in keeping with this.
Secondary adrenal insufficiency would present very similarly to Addison's disease. However, secondary causes are not typically associated with hyperpigmented palmar creases.
Tuberculosis can be a cause of adrenal insufficiency, however, there is nothing in the history given here to suggest this.
A 56-year-old female is brought in to A&E by her husband. He is concerned as she has been vomiting and complaining of abdominal pain. You are unable to get a history from the patient because she is agitated and confused. Her husband reports that this is completely different from her baseline mental state. She has been off work for a few days because she has been feeling 'generally unwell'. He thinks her GP gave her some antibiotics for a 'water infection'.
Her husband tells you that she has a past medical history of a 'thyroid problem' and says it has affected her eyes. He also tells you that she has high blood pressure. He is unsure what medications she is on.
On examination, she is jaundiced and has exophthalmos.
Her heart rate is 180 and is irregularly irregular. Her temperature is 41 degrees Celsius. Her blood pressure is 95/75.
You perform an ABCDE assessment and begin resuscitation as appropriate.
Which of the following is the most important medication to commence?
E) Carbimazole
This patient is having a hyperthyroid crisis or 'thyrotoxic storm'. This is suggested by her past medical history of Graves' disease (thyroid problem with exophthalmos). Hyperthyroid crisis classically occurs in patients with underlying Graves' disease or toxic multinodular goitre. The presentation can be varied but it often presents as sudden onset severe hyperthyroidism with hyperpyrexia (over 41 degrees), tachycardia (with or without atrial fibrillation), abdominal symptoms and altered consciousness level. It can be precipitated by acute illness, non-compliance with antithyroid medication and trauma, including surgery. Management includes resuscitation (oxygen and fluids), antithyroid treatment (carbimazole or propylthiouracil), IV beta blockers and hydrocortisone. After 4 hours, aqueous iodine oral solution should be administered.
Diclofenac is given acutely for pain control in conditions such as renal colic. This typically presents with severe, loin to groin pain.
Insulin would be used in diabetic ketoacidosis (DKA). This condition can present in a similar way with gastrointestinal upset and altered mental state, but it would not typically feature hyperpyrexia.
Procyclidine hydrochloride is used to manage acute dystonias such as oculogyric crisis. An oculogyric crisis can be caused by many drugs but classically neuroleptic medications and metoclopramide in young females.
Levothyroxine is used intravenously to treat myxoedema coma.
A 65-year-old man presents to his GP with vision loss, headaches and cold intolerance. He also mentions he has unintentionally gained 5kg in the last 3 months. On examination, he is found to have a bitemporal hemianopia and a pulse of 50 bpm. A thyroid function test is ordered. Which of the following test results is most likely to belong to this patient?
D) TSH: Low T4: Low T3: Low
This man most likely has a pituitary adenoma causing secondary hypothyroidism. In addition to thyroid function tests, he would also need an MRI of the head to confirm the presence of an adenoma. His thyroid function tests would be expected to show TSH: Low T4: Low T3: Low. In this case secretion of TSH is most likely suppressed due to compression of TSH secreting cells (thyrotrophs) by a pituitary mass.
TSH: high T4: low T3: low- This represents primary hypothyroidism ('primary' meaning that the pathology is at the level of the thyroid), which is a possibility and is in fact far more common than secondary hypothyroidism. However, the bitemporal hemianopia and history of headaches suggest that a pituitary mass is causing this man's symptoms.
TSH: low T4: high T3: high- This represents primary hyperthyroidism, which would not explain this man's symptoms of cold intolerance and weight gain.
TSH: high T4: normal T3: normal- This represents subclinical hypothyroidism, which would not cause such marked symptoms of hypothyroidism as are present in this man.
Sarah, aged 26, presents to her GP wanting advice about conception. Specifically, she is wondering when the best time to have sexual intercourse would be in order to maximise her chances of becoming pregnant. Sarah has a 28-day regular menstrual cycle.
At which point in her menstrual cycle will the patient be most fertile?
E) Days 9-16
Most women ovulate in the middle of their cycle, at around day 14 in a 28-day cycle, in response to a surge in luteinizing hormone. This corresponds to the time where conception is more likely to occur. General advice for patients trying to conceive would be to have regular, unprotected sexual intercourse every 2-3 days.
A 24-year-old woman is admitted to the hospital having been referred by her GP due to extremely low BMI. She has a history of anorexia nervosa and hasn't been eating as much for the last couple of months. On examination, she has a BMI of 14.0kg/m^2.
Some patients, such as those with severe eating disorders, are at risk of refeeding syndrome.
Given this patient's history, which of the following electrolyte imbalances would suggest that she is at risk of refeeding syndrome?
C) Hypophosphatemia
As the name "Refeeding" syndrome suggests, it is an abnormality that presents when a person is fed following a period of starvation.
Hypophosphataemia is the hallmark of the syndrome and predominant cause of the refeeding syndrome. When the patient has a low nutritional intake, phosphate stores are depleted. Abrupt carbohydrate intake from "refeeding" triggers insulin release, which causes cellular uptake of phosphate (and other electrolytes), in turn causing a decrease in serum phosphate levels. Insulin also triggers the cells to make phosphate-requiring molecules (e.g. adenosine triphosphate and 2,3-diphosphoglycerate), further depleting the body of phosphate.
Patients undergoing chemotherapy, malnourished elderly patients, post-operative patients and individuals dependent on the care of others are also potentially at risk of refeeding syndrome.
A 17-year-old woman was seen in clinic with a history of excessive growth of body hair, needing to epilate every 2 weeks. She first noticed this problem when she was 15 and this has become worse over the years. She is particularly worried about the appearance of facial hair. Her menarche was at 13 years and her periods last 7-8 days every 30-35 days. She says her periods are not painful or heavy and she is not sexually active. She denies noticing any changes to her voice and she does not take any regular medications.
Clinical examination reveals:
BMI of 28 kg/m2
Hirsutism affecting the chest, abdomen and upper lip
Blood pressure normal
Further investigations reveal:
Normal pelvic ultrasound
LH level 7 IU/L
FSH level 6 IU/L
Serum testosterone level 2.1 nmol/l
SHBG 21 nmol/L
Prolactin and TSH levels normal
What is the most likely diagnosis?
B) Idiopathic hirsutism
Hirsutism is defined as the excessive growth of thick dark hair in an androgen-dependent pattern where hair growth in women is usually minimal or absent - eg, the face, the chest and the areolae. It occurs as a result of increased androgen production, increased skin sensitivity to androgens, or both.
Idiopathic hirsutism is common and often familial. It often starts after puberty and progresses slowly. There are typically no other signs of virilism. Menstrual function and investigations are normal. It is a diagnosis of exclusion and thought to be related to disorders in peripheral androgen activity.
Polycystic ovary syndrome (PCOS) is one of the commonest causes of hirsutism, however, it is unlikely in this case given the normal menstrual cycle and abdominal ultrasound scan. PCOS typically presents with oligomenorrhoea (defined as <9 periods per year), infertility, acne, hirsutism and obesity.
Cushing's syndrome typically presents with weight gain, easy bruising, "moon face", striae and proximal muscle weakness. Hirsutism is also a feature of Cushing's syndrome, however, the absence of other typical cushingoid features makes the diagnosis less likely.
Drug-induced hirsutism is highly unlikely given the patient reports not taking any regular medications. Typical causative agents include glucocorticoids, danazol, sodium valproate and androgenic progestogens.
Androgen-producing ovarian tumors (e.g. luteoma of pregnancy, Leydig cell tumours, hilar cell tumors) present with hirsutism, raised testosterone levels and signs of virilism (differentiating it from idiopathic hirsutism).
Signs of virilism can include:
Acne
Alopecia
Truncal obesity
Clitoromegaly
Deepening of voice
Increased muscle mass
Malodorous perspiration
Infertility
Menstrual dysfunction
A man comes into your practice complaining of weight gain, fatigue, constipation and low mood. Examination reveals a large painless goitre on the man's neck. You send for a thyroid function test. The test results reveal:
TSH: 15 mU/L
Free T3: 2.4 pmol/L
Free T4: 5 pmol/L
Thyroid antibody tests are also ordered which show:
Anti-thyroglobulin antibodies - present.
Anti-thyroid peroxidase antibodies - present.
Which of the following is the most likely diagnosis?
A) Hashimot's thyroiditis
Based on the test results we know that the patient has hypothyroidism. This can be inferred from the elevated levels of TSH, yet low levels of free T3 and free T4. This suggests a primary cause of hypothyroidism as the thyroid gland isn't responding to the stimulus (TSH) to secrete FT3 and FT4.
The presence of the anti-thyroglobulin and anti-thyroid peroxidase antibodies suggests an autoimmune condition such as Hashimoto's thyroiditis. This generally presents as hypothyroidism and is therefore consistent with the signs and symptoms observed.
Notes on thyroid hormone physiology:
1. Thyroglobulin is a precursor of the thyroid hormones T3 and T4.
2. T4 is far more abundant than T3. However, T3 is far more active, as such peripherally T4 is often converted to T3 and rT3 (reverse T3, inactive).
3. This is relevant as various conditions might skew such concentrations.
4. The T3 to rT3 ratio is usually equal, however in pregnancy, liver/renal failure, fasting, beta-blockers etc we can see this shift towards rT3.
Toby Robertson is a 15-year-old boy who attends the GP practice with his mother who is concerned that he appears to be developing breasts. He has no significant past medical history but he does receive support at school for learning difficulties.
Findings on examination include:
Tall stature
Elongated arms and legs
Reduced facial and body hair
Wide hips
Gynaecomastia
Small testicles
Which of the following is the most likely diagnosis?
A) Klinefelter's syndrome
The symptoms in unison point towards Klinefelter's syndrome also known as 47XXY syndrome where a male has an extra copy of the X chromosome (XXY). Those with Klinefelter's syndrome have a number of characteristic features including tall stature, sparse facial/body hair, gynecomastia, infertility, learning difficulties and small testicles (due to hypogonadism).
Turner syndrome can be defined as loss or abnormality of the second X chromosome in at least one cell line in a phenotypic female. Clinical features are vary widely but can include pubertal delay, ovarian failure, learning difficulties, neck webbing, low set ears and epicanthic folds.
Fragile X syndrome is a repeat DNA expansion disorder. Typical features include learning difficulties, delayed developmental milestones, high forehead, large testicles, facial asymmetry, large jaw and long ears.
Familial male-limited precocious puberty is a form of gonadotropin-independent precocious puberty in which boys experience early onset of puberty, sometimes from the age of 1 year old. It is an autosomal dominant condition caused by a mutation in the LH receptor leading to the production of large quantities of sex steroids. In addition to all the symptoms of puberty, the patient may also have a shortened spinal length due to premature epiphyseal
maturation.
A 54-year-old male who was previously well presents to his GP with ongoing headaches for the past few months. He also complains of difficulty sleeping at night due to breathlessness. He reports struggling to fit into his clothes recently and his wedding ring no longer fits. His blood pressure in clinic is 162/83 mmHg.
What is the single most-appropriate test that would confirm the likely diagnosis?
A) Insulin-like growth factor 1
Acromegaly is a manifestation of excessive growth hormone, which is produced by the anterior pituitary gland. The most common cause for excessive production of growth hormone is a pituitary adenoma, a benign tumour of the pituitary gland. Growth hormone stimulates the liver to produce IGF-1.
This results in several signs and symptoms:
1. Structural and bone changes, e.g., protruding jaw, large ears/nose, forehead "bossing" (anteriorly protruding), etc.
2. Sleep apnea
3. Galactorrhea
4. Headache
5. Sweating
6. Hypertension
7. Carpal tunnel syndrome
8. Erectile dysfunction
9. Gigantism, e.g., clothes not fitting anymore, enlarged fingers etc.
Treatment can consist of medical management with drugs to block growth hormone or surgical involvement to remove tumors.
The other options are all used to identify other endocrinological conditions.
1. Aldosterone-renin ratio is used to investigate hyperaldosteronism (e.g. Conn's syndrome).
2. Short-acting synacthen test is important in hypoaldosteronism (e.g. Addison's disease).
3. Dexamethasone suppression test is used when investigating suspected Cushing's syndrome/disease.
4. Urinary catecholamines are used in the work-up of pheochromocytoma.
A 43-year-old female presents with central weight gain, purple striae on the abdomen, proximal muscle weakness and male pattern facial hair growth. After further investigation, the patient is found to have a functioning microadenoma in the pituitary fossa.
Which of the following are the most likely findings?
B) Hyperglycemia and raise ACTH
Cushing's disease is a specific name given to a pituitary adenoma that causes Cushing's syndrome (hypercortisolism). The adenoma secretes ACTH (adrenocorticotropic hormone) that stimulates the zona fasciculata in the adrenal gland to secrete cortisol. One of the systemic effects of cortisol is to raise blood glucose. The dexamethasone suppression test is used to aid diagnosis. The test involves giving a low (usually 1–2 mg) and high (8 mg) dose of dexamethasone and measuring the levels of cortisol after each. A low dose of dexamethasone suppresses cortisol in individuals with no pathology in endogenous cortisol production. A high dose of dexamethasone exerts negative feedback on pituitary neoplastic ACTH-producing cells (Cushing's disease), but not on ectopic ACTH-producing cells or adrenal adenoma (Cushing's syndrome).
The male pattern hair growth in females is known as hirsutism and is due to excess androgens present in Cushing's, therefore, hyperandrogenism will be present in patients. The main differential diagnosis in females with hyperandrogenism is PCOS (polycystic ovary syndrome) where the ovaries develop cysts, become dysfunctional and secrete testosterone.
Linda, a 54-year-old woman, has presented to her GP with recurrent occurrences of dizziness that occur in the evenings before tea time. She also begins to feel faint and her hands start to shake during these episodes.
She has a history of anxiety, hypertension and type 2 diabetes mellitus.
Her current medications include Ramipril, Metformin and Gliclazide.
On examination:
There is a high BMI but no other abnormality.
Sitting blood pressure 136/84
Standing blood pressure 128/78
The patient's complaints of dizziness, light-headedness and shaking are common symptoms of hypoglycemia. The timing is also indicative (before her evening meal, when her blood sugars are at their lowest). Why is she having these episodes? To answer this needs knowledge of the mechanism and side effects of diabetes medication (primarily the difference between insulin sensitisers and insulin secretagogues).
Gliclazide is an insulin secretagogue, increasing the amount of endogenous insulin produced. Sometimes too much Insulin can be secreted, resulting in hypoglycemia.
Metformin is an insulin sensitizer and therefore makes the amount of insulin in the body more effective. While metformin reduces average blood sugars it very rarely causes hypoglycemia. Therefore reducing the offending drug, gliclazide, is the most appropriate management option.
Other causes of her symptoms like postural hypotension are unlikely given the absence of a drop in blood pressure on standing (needs to be >20 mmHg systolic or >30 mmHg systolic in a patient with hypertension). Whilst the shaking could be a sign of panic attacks there is nothing in the stem to suggest a trigger or psychological component. Therefore the two management options for those conditions (fludrocortisone and propanolol respectively) are not appropriate.
Her HbA1c of 49, while diagnostic for diabetes, suggests that her average blood sugar is around 7.9. This could either mean very well controlled disease or in this case; a large range of blood sugars from hypoglycaemia to hyperglycemia with a falsely reassuring average blood sugar.
A 60-year-old male presents to general practice with several weeks' history of headaches, worse in the morning and on bending forwards or coughing. He has been nauseous and increasingly clumsy, knocking glasses off the dining table with his elbows and bumping his head on cupboard doors. He is embarrassed to report erectile dysfunction and discharge from both nipples. You note that he has a markedly thick-set brow and very prominent jaw, and when you shake his hand you notice it engulfs yours. On examining his cranial nerves you find a visual field defect to account for his recent clumsiness.
Blood tests reveal a prolactin level of 80,000 mIU/L (normal range <~400mIU/L depending on the laboratory.
Which one of the following visual field defects is this gentleman most likely to suffer?
A) Bitemporal hemianopia
This patient has a pituitary macroadenoma as suggested by his symptoms of raised intracranial pressure, galactorrhoea and acromegaly. The anterior pituitary gland is responsible for secretion of prolactin, luteinizing hormone, follicle stimulating hormone, growth hormone, thyroid stimulating hormone and adrenocorticotropic releasing hormone in response to stimulation by the hypothalamus. The posterior pituitary secretes vasopressin and oxytocin.
In health, the left lateral (temporal) side of the visual field is represented upon the left medial (nasal) aspect of the retina, on account of the convex shape of the lens. The right temporal aspect of the visual field is represented on the right nasal aspect of the retina, and the median of the visual field represented on the temporal aspects of the retinas.
Sensory information is conveyed along the optic nerves to the optic chiasma, where both nerves meet above the bony sella turcica (Turkish Saddle), in which the pituitary gland may be found. The fibres from the temporal aspects of each retina continue beyond the chiasma on the ipsilateral sides of the brain, but the fibres from the nasal aspects of each retina cross the chiasma to the contralateral side and are consequently at risk of compression from pituitary or other suprasellar tumours, as in this case.
Beyond the chiasma, second-order neurons carry sensory information to the lateral geniculate nuclei, synapsing with third-order neurons which ultimately terminate in the occipital cortex where the complete image is formed.
A 40-year-old male presents to his GP with a four-month history of episodic sweating, palpitations, anxiety and headaches. He says these episodes begin and resolve suddenly, and "usually last a few minutes". He states that his wife reports that he looks pale when these happen. He cannot identify any triggers, and states they often occur at rest.
He has no significant past medical history and is on no regular medication. He is a non-smoker, non-drinker and denies illicit drug use. He has no family history of note, including no family history of neoplastic or endocrine disease. He is married and works as a shopkeeper.
General inspection is unremarkable and vital signs are within normal limits.
Cardiovascular, respiratory, abdominal and thyroid examinations are all normal. Of note, fundoscopy reveals bilateral AV nipping, cotton wool spots and flame haemorrhages, but no papilloedema.
Which of the following investigations is most likely to reveal the underlying diagnosis?
A 40-year-old male presents to his GP with a four-month history of episodic sweating, palpitations, anxiety and headaches. He says these episodes begin and resolve suddenly, and "usually last a few minutes". He states that his wife reports that he looks pale when these happen. He cannot identify any triggers, and states they often occur at rest.
He has no significant past medical history and is on no regular medication. He is a non-smoker, non-drinker and denies illicit drug use. He has no family history of note, including no family history of neoplastic or endocrine disease. He is married and works as a shopkeeper.
General inspection is unremarkable and vital signs are within normal limits.
Cardiovascular, respiratory, abdominal and thyroid examinations are all normal. Of note, fundoscopy reveals bilateral AV nipping, cotton wool spots and flame haemorrhages, but no papilloedema.
Which of the following investigations is most likely to reveal the underlying diagnosis?
A) Dexamethasone suppression test
B) 24-hour urinary catecholamines
C) Renin-aldosterone ration
D) Thyroid function tests
E) 24-hour urinary 5-HIAA
B) 24-hour urinary catecholamines
The most likely underlying diagnosis in this patient is pheochromocytoma. This is suggested by the episodic constellation of symptoms (sweating, palpitations, anxiety, pallor, headache) and evidence of hypertensive retinopathy. 90% of cases are sporadic, hence the lack of positive family history. 24-hour urinary catecholamines is an investigation for pheochromocytoma, therefore is the most likely test of the options to reveal this diagnosis.
Thyroid function tests represent a simple and important investigation to perform in this patient to rule out thyrotoxicosis. This diagnosis is less likely, however, given the episodic nature of his symptoms and normal thyroid exam.
24-hour urinary 5-HIAA is an investigation for carcinoid syndrome. This is less likely as it is characterized by wheeze, dry skin flushing, diarrhoea and evidence of tricuspid regurgitation/pulmonary stenosis.
Renin:aldosterone ratio is an investigation test for primary hyperaldosteronism (e.g., Conn's syndrome). This is less likely as it is often asymptomatic or presents with symptoms of hypokalaemia (e.g., muscle weakness, cramps).
Dexamethasone suppression test is an investigation for Cushing's syndrome. This is less likely given the absence of Cushingoid features.
Mrs. White is a 57-year-old lady on your ward who had a total thyroidectomy three days ago for non-metastatic thyroid cancer. Unfortunately, the surgeon was unable to preserve the parathyroid glands due to local invasion of the tumour, however, at this stage, it is believed that the operation was curative. She has a background of hypertension, chronic kidney disease and asthma.
Over the last few hours, the nursing staff have noticed that she has been slightly disoriented, complaining of muscle cramps and colicky abdominal pain and is getting anxious and irritable. Her vitals are stable.
You suspect there may be an electrolyte abnormality.
As you arrive at the bedside the nurse is inflating the blood pressure cuff which causes Mrs White's hand gradually flexes at the wrist and MCP joints until the cuff deflates.
Which clinical sign does this demonstrate?
E) Trousseau's sign
Mrs White's history, clinical signs and test results indicate she has hypocalcaemia most likely due to hypoparathyroidism. Other causes of hypocalcaemia include chronic kidney disease, pseudohypoparathyroidism, acute rhabdomyolysis, vitamin D deficiency, hypomagnesaemia, osteomalacia, acute pancreatitis, over-hydration, respiratory alkalosis.
The clinical features can be remembered with the mnemonic SPASMODIC
Spasms (e.g. Trousseau's Sign)
Perioral paraesthesia
Anxious, irritable, irrational
Seizures
Muscle tone increase in smooth muscle (wheeze, dysphagia, colic)
Orientation impairment, confusion
Dermatitis
Impetigo herpetiformis
Chvostek's Sign
The other choices are incorrect because:
1. Chvostek's Sign - twitch of the corner of the mouth on tapping the facial nerve over the parotid. This sign also suggests hypocalcemia, but is incorrect as the sign being demonstrated is Trousseau's sign.
2. Rosving's Sign - palpation of left iliac fossa producing pain in the right iliac fossa. This is also known as cross tenderness, the presence of this sign suggests appendicitis.
3. Babinski Sign - dorsiflexion of toes on stimulating the sole of the foot. This suggests an upper motor neuron lesion.
4. Troisier's Sign - enlarged left supraclavicular lymph node (Virchow's Node). This suggests metastatic abdominal malignancy.
Acanthosis nigricans is associated with which of the following conditions?
A) Polycystic ovarian syndrome
Acanthosis nigricans is characterized by symmetrical, dark, 'velvety' plaques which arise on the neck, axillae and groin creases in conditions associated with raised insulin, such as:
1. Obesity
2. Type 2 diabetes
3. Polycystic ovarian syndrome
4. Cushing's disease
A 40-year old male patient presents with orthostatic hypotensive episodes associated with dizziness on standing that has been noted at home with his own blood pressure monitoring. On further questioning, you elicit that he has pulsatile headaches and experiences frequent palpitations with episodes of sweating. His past medical history is not significant, and he has no known allergies. His paternal family history is significant for neck cancer, but he cannot recall which type of cancer. The remainder of his history is unremarkable. You order abdominal imaging that shows a focal abnormality on the left side of the abdomen and you refer the patient to a general surgeon specialising in endocrine disease.
Which vein must be clamped prior to removing this mass?
C) Left suprarenal (adrenal) vein
This is a pheochromocytoma and the left suprarenal (adrenal) vein must be clamped prior to excising this tumor. The clamped suprarenal vein is always ipsilateral to the side of the pheochromocytoma.
Pheochromocytoma is a tumor the adrenal medulla that is metabolically active and secretes catecholamines - adrenaline and noradrenaline. These catecholamines are sympathetic activators and cause the 3 P's - pulsatile headaches, palpitations and perspiration. However, it is not uncommon for the first presentation of pheochromocytoma to be orthostatic hypotension, which may manifest as dizziness on standing. By clamping the left suprarenal vein, a catecholamine surge is prevented, which may be fatal in patients undergoing resection of this tumor.
In this question, it would not be relevant to clamp the right suprarenal vein, as the right suprarenal gland is most likely healthy. The renal veins should not be clamped, as a venous infarct may occur in the healthy kidneys. The right lumbar vein does not contribute to the venous drainage of the suprarenal glands and is incorrect.
A 45-year-old lady presents to her GP with weight gain, sleepiness and constipation of 4 months duration. The GP orders a basic set of blood tests including thyroid function tests:
TSH: 5.1 mU/L (0.4-4.0 mU/L)
T4: 2.0 pmol/L (9-24 pmol/L)
T3: 0.9 pmol/L (3.5-7.8 pmol/L)
The presence of which antibody in this patient's serum would be most useful in establishing a definitive diagnosis?
B) Anti-thyroid peroxidase antibody (Anti-TPO)
This lady has primary hypothyroidism based on her thyroid function tests (TFTs) which show a raised TSH but a low T3/T4.
Anti-thyroid peroxidase antibodies (Anti-TPO) are found in 90-95% of patients with autoimmune thyroiditis (the most common cause of primary hypothyroidism in the UK and North America). This antibody is, therefore, most useful for establishing a definitive diagnosis of autoimmune thyroiditis. Anti-TPO antibodies are also present in about 75% of cases of Graves' hyperthyroidism and can help to differentiate autoimmune hyperthyroid disease from toxic nodular hyperthyroidism.
TSH-receptor antibodies are commonly present in Graves' disease (the most common cause of primary hyperthyroidism in the UK). They have been shown to have a sensitivity of 98% and a specificity of 99%. This test is not available in all clinical settings, meaning clinical findings and other antibodies need to be used together to make a diagnosis.
Anti-thyroglobulin antibodies are often found in patients with Hashimoto's thyroiditis or Graves' disease. Their presence is of limited use in the diagnosis of these diseases, since they may also be present in healthy euthyroid individuals.
Anti-nuclear antibodies (ANA) can be a marker of an autoimmune process, but they are fairly non-specific and are associated with several autoimmune disorders. ANA is most commonly associated with systemic lupus erythematosus (SLE).
Rheumatoid factor is associated with rheumatoid arthritis (present in about 80% of cases) and Sjögren's syndrome (present in 70% of cases). It is also found in a wide range of other autoimmune conditions and therefore it's diagnostic value as a single test is fairly poor.
A 66-year-old woman attends her follow up appointment 6 months after a total thyroidectomy for locally invasive medullary thyroid cancer.
Past medical history: medullary thyroid cancer, hypertension and osteoarthritis of the left hip
On examination, her wound is well healed and no surgical complications evident. She is clinically euthyroid and there is no palpable lymphadenopathy.
Medications: Levothyroxine (replacement dose) and Amlodipine
Given her medical history, which of the following blood tests would be most useful to detect recurrence of malignancy?
E) Calcitonin
Medullary thyroid cancer is a malignancy of the parafollicular cells (C Cells). These cells are not involved in the production of thyroid hormones, but release calcitonin which is involved in calcium homeostasis. The treatment of medullary thyroid cancer is generally surgical as C cells are not responsive to radioactive iodine therapy. The most appropriate test to detect recurrence is to detect an increase in calcitonin levels which suggests the proliferation of C cells and likely progression/return of the disease.
T4 should be measured post thyroidectomy to ensure that the patient is euthyroid and that the dose of replacement Levothyroxine is adequate. However, this is not a useful test to determine disease recurrence.
Thyroglobulin is a protein produced in the follicular cells of the thyroid gland and is an important component of thyroid hormones. Thyroglobulin is a useful marker of recurrence in malignancies affecting the follicular cells themselves e.g. follicular and papillary carcinoma. However, as medullary thyroid cancer doesn't affect the follicular cells, thyroglobulin is not used for follow up.
TSH has no effect on parafollicular cells (not TSH responsive) as C cells secrete in response to calcium levels. In cancers affecting the follicular cells, it is important that TSH levels are measured to ensure they are adequately suppressed to prevent a recurrence, however, this is not necessary for medullary thyroid cancer.
T3: T4 is converted to T3 in the peripheral tissues and it is the active form of the hormone, impacting many cellular functions. Like T4, it may be useful to assess the thyroid status of the patient/ direct further thyroid hormone replacement but it is not useful to detect recurrence.
A 67-year-old lady presents to her GP with weight loss, diarrhea, and increased sweating of 2 months duration. The GP orders a basic set of blood tests including thyroid function tests:
TSH: 0.1 mU/L
T4: 29 pmol/L
T3: 8.1 pmol/L
The presence of which antibody in this patient's serum would be most useful in establishing a definitive diagnosis?
B) TSH receptor antibodies
This lady has primary hyperthyroidism as shown by her elevated T3/T4 and suppressed TSH.
TSH-receptor antibodies are commonly present in Graves' disease (the most common cause of primary hyperthyroidism in the UK). They have been shown to have a sensitivity of 98% and a specificity of 99% so the presence of this antibody would be the most useful in establishing a definitive diagnosis. This test is not available in all clinical settings, meaning clinical findings and other antibodies need to be used together to make a diagnosis.
Anti-thyroid peroxidase antibodies are present in about 75% of cases of Graves' hyperthyroidism and can help to differentiate autoimmune disease from toxic nodular hyperthyroidism. Anti-thyroid peroxidase antibodies are also found in 90-95% of patients with autoimmune thyroiditis. This antibody is, therefore, more useful for establishing a definitive diagnosis of autoimmune thyroiditis.
Anti-thyroglobulin antibodies are often found in patients with Hashimoto's thyroiditis or Graves' disease. Their presence is of limited use in the diagnosis of these diseases, since they may also be present in healthy euthyroid individuals.
Anti-nuclear antibodies (ANA) can be a marker of an autoimmune process, but they are fairly non-specific and are associated with several autoimmune disorders. ANA is most commonly associated with systemic lupus erythematosus (SLE).
Rheumatoid factor is associated with rheumatoid arthritis (present in about 80% of cases) and Sjögren's syndrome (present in 70% of cases). It is also found in a wide range of other autoimmune conditions and therefore it's diagnostic value as a single test is fairly poor.
John Smith, a 55-year-old accountant with type 2 diabetes mellitus (T2DM), obesity and hyperchloesterolaemia, presents to Diabetes Clinic for review. HiS medications include metformin 1 g BD and atorvastatin 20 mg daily. He has no known drug allergies.
On today's review, he is asymptomatic. He reports full adherence with medication and lifestyle interventions.
On physical examination, his body mass index (BMI) is 31 kg/m2. Fundoscopy and lower limb neurological examination are both unremarkable.
His HbA1c is 60 mmol/mol. Urine dipstick and renal profile are both normal.
His Consultant decides to augment his metformin with canagliflozin.
What should his target HbA1c be?
D) 53 mmol/mol
This patient is on dual therapy (metformin and canagliflozin), so a target of 53 mmol/mol is most appropriate.
Patients with type 2 diabetes mellitus (T2DM) should be involved in decision-making around target HbA1c. In general, a relaxed target HbA1c of 53 mmol/mol is appropriate in the following situations:
1. If the patient is on a single agent associated with hypoglycemia (e.g., sulfonylureas)
2. If the patient is on dual/triple therapy
3. If the patient is at higher risk of the consequences of hypoglycemia (e.g., driving/operating machinery as part of occupation).
A 50-year old male with type 2 diabetes mellitus, currently on metformin 1g TDS, attends the endocrinology clinic for review. His HbA1c is 58mmol/mol (7.5%). His endocrinologist elects to add empagliflozin.
Which of the following classes does empagliflozin belong to?
B) SGLT-2 inhibitor
Empagliflozin is an SGLT-2 inhibitor, which causes decreased renal glucose reabsorption and increased elimination of glucose in the urine.
Common adverse effects include increased urination, thirst, and urinary tract and genital infections (eg. balanitis). They are associated, in rare cases, with DKA and necrotizing fasciitis of the perineum (Fournier's gangrene).
Other examples of SGLT-2 inhibitors include canagliflozin and dapagliflozin.
DPP-4 inhibitors include the '-gliptins' eg. sitagliptin. They increase incretin levels, which inhibits glucagon secretion.
GLP-1 agonists such as dulaglutide, semaglutide and exenatide, are given subcutaneously. They are mimetics of incretin, so inhibit glucagon secretion.
Metformin is a biguanide. It is usually first-line in the management of type 2 diabetes mellitus and increases insulin sensitivity to decrease hepatic gluconeogenesis.
The sulfonylureas stimulate pancreatic beta cells to secrete insulin. Gliclazide, glipizide and tolbutamide are examples of these.
Which of the following is not a cause of pitting edema?
C) Grave's disease
Grave's disease, and other causes of hyperthyroidism, can present with pretibial myxedema. This is discolored, non-pitting swelling of the lower legs, usually found bilaterally and can be sore or itchy. It is a result of excess glycosaminoglycans and mucin deposits in the dermis of the skin.
Congestive cardiac failure and venous insufficiency cause pitting edema due to impaired transport of blood around the body. Low serum albumin allows for leakage of water into tissues, causing pitting edema. Pregnancy is another known cause of pitting edema, due to increased cardiac demand and pressure of the uterus on the pelvic veins and inferior vena cava.
You are a Junior Doctor working in General Practice (GP). During your clinic, you see a 72-year-old man who is complaining of feeling increasingly tired over the last 3 months. His wife tells you that he has been very disorientated and she has had enough of his ‘brain fog'. On taking a thorough history you learn that he has been urinating much more than normal, and has been increasingly thirsty. He tells you that this has all been going on for about 3 months.
On clinical examination, you find that he has reduced skin turgor and dry mucous membranes. He is also tachycardic (heart rate 140 beats per minute), hypotensive (blood pressure 84/56). A respiratory examination reveals clear chest auscultation.
Urinalysis shows dilute and slightly straw-coloured urine, with very small glucose level and free from ketones, nitrites, proteins, neutrophils and blood cells.
You request some blood tests which demonstrate the following:
Urea: 20 (raised)
Creatinine 92 (normal)
Sodium 158 (raised)
Fasting Blood Glucose 5.2 (normal)
Which of the following is the most likely diagnosis for this patient?
E) Diabetes insipidus
Diabetes insipidus (DI) is a condition characterised by either a deficiency of ADH (neurogenic) or an insensitivity to ADH (nephrogenic).
ADH is produced in the posterior pituitary in response to increased blood osmolality and tonicity or decreased blood volume. It specifically acts on the distal convoluted tubules (DCT) and collecting ducts (CD), where it regulates urine volume and osmolarity. DI in adults is usually characterized by polydipsia, polyuria, and nocturia. In this case, there is chronic dehydration, hypovolemia and hypernatremia with water being excreted disproportionately to the electrolytes (especially sodium). Hence the dilute urine, and signs of dehydration: fatigue, reduced skin turgor, dry mucous membrane, low blood pressure and fast heart rate. Electrolyte imbalance may have contributed to the confusion (‘brain fog’), weakness and tremors.
In the syndrome of inappropriate anti-diuretic hormone (SIADH), there is too much ADH production and/or action, resulting in reduced urine output. This is characterized by hyponatraemia, secondary to the effects of chronic water retention.
Type-2 diabetes mellitus can also present with polyuria and polydipsia. However, these patients will present with higher fasting blood glucose level and increased glucosuria.
Addison’s disease (caused by adrenal insufficiency/low production of aldosterone, cortisol, etc.) whilst also sharing several similar symptoms with DI, might present with some other symptoms including weight loss, hyperpigmentation, postural dizziness and syncope, etc.
Liver cirrhosis will often present with dark urine, jaundice, skin bruises and several gastrointestinal symptoms.
Diabetes insipidus typically involves the production of more than ____ L of urine in a 24-hour period.
B) 3
A 61-year-old man presents to A&E with severe lethargy, weight loss (22 kg over the past three weeks), polyuria and a sore mouth on a background of poorly controlled type 2 diabetes mellitus.
Bedside point of care testing includes venous blood gas (VBG) analysis, showing the following results:
pH: 7.38
lactate: 4.4 mmol/L
glucose: >27.8 mmol/L (the upper limit of testing on VBG equipment)
sodium (Na+): 148 mmol/L
potassium (K+): 5.1 mmol/L
haemoglobin (Hb): 171g/L
capillary ketones: 1.0 mmol/L.
A diagnosis of hyperosmolar hyperglycemic state (HHS) is given.
What is the patient's serum osmolality?
A) 350
B) 320
C) 370
D) 295
E) 280
C) 370
The diagnosis of hyperosmolar hyperglycemic state (HHS) is confirmed by calculating serum osmolality according to the following formula: 2(Na+) + urea + glucose. In this patient, the calculation would be: 2(148) + 27.4 + 46.6 = 370.
Normal osmolality is 280-295, with values above 320 suggestive of HHS. Osmolality correlates with level of consciousness, where higher osmolality levels correspond to greater impairment.
Treatment is with aggressive IV fluids, with a typical positive balance of 3-6 litres over 12 hours. Anticoagulation is also an important aspect of management, as the patient's profoundly dehydrated state predisposes them to clotting.
It is important to note that the mortality rate of HHS is estimated to be between 5% and 20%, compared to diabetic ketoacidosis (DKA), which is estimated to be 1-5%. Capillary ketones of >3 mmol are a requirement for a diagnosis of DKA.
Whilst on clinical rotation, a consultant nephrologist informs you of a 21-year old female patient – a professional marathon runner – who is found to have mucosal dryness, reduced jugular venous pulse, and reduced skin turgor. Blood tests reveal a low plasma sodium concentration of 120 mmol/l, and her urine sodium concentration is elevated at 28 mmol/l.
The patient’s vital signs include a blood pressure of 82/50 (mmHg) and a heart rate of 132 bpm, with the rest being unremarkable. The patient has been complaining of fatigue, confusion and muscle cramps.
The consultant asks you for your differential diagnosis given the patient’s profile. Which group of conditions below would most likely be included in your list?
A) Diuretics Use, Addison’s Disease, Salt-Wasting Nephropathy
The clinical vignette describes a patient who is suffering from hypovolemia (i.e. dehydrated) and hyponatremia, together with a confirmed increase in sodium loss in urine. The three aforementioned conditions in the correct answer can cause this presentation pattern. Note that drinking copious amounts of water (as a professional marathon runner) might also result in hyponatremia.
The presentation patterns and signs and symptoms of conditions listed in the other options would be different. For instance, diabetes mellitus will usually present with glucosuria, glomerulonephritis might present with hematuria, whilst pyelonephritis (infection and inflammation of the kidneys indicating upper UTI), would present with flank, back and groin pain, fever, nausea and vomiting, etc. In psychogenic polydipsia, hypothyroidism and SIADH, the patient would usually be euvolemic (not hypovolemic), whilst in heart failure, cirrhosis and nephrotic syndrome, patients are usually edematous (i.e. in a hypervolemic state).
A 54-year-old female presents to her GP with unintentional weight loss of 6kg over 6 months. She also reports a change in her bowel habit, with increasing frequency and diarrhea over the last two months. She denies any abdominal pain, melaena, nausea or vomiting, but does report feeling "hot all the time" and occasionally anxious. She has no past medical history and takes no regular medication. She denies smoking or drinking alcohol.
On examination, her pulse is 106bpm and irregular with a pronounced peripheral tremor. Her abdomen is soft and non-tender, with no organomegaly and normal bowel sounds. Rectal examination is normal. An ECG reveals atrial fibrillation.
What is the most likely cause of her weight loss?
E) Hyperthyroidism
The most likely diagnosis is hyperthyroidism given the history of unintentional weight loss, diarrhea, heat intolerance and anxiety in combination with the clinical findings of new-onset atrial fibrillation and peripheral tremor. A focused thyroid status examination may have revealed a goitre, thyroid-related eye disease and proximal muscle weakness.
Gastrointestinal malignancy should be considered given the unintentional weight loss and change in bowel habit, however, the absence of blood in the stool and normal abdominal examination are somewhat reassuring. Heat intolerance, peripheral tremor and atrial fibrillation are not associated with gastrointestinal malignancy.
Coeliac disease can cause weight loss and change in bowel habit, however, it would not be associated with heat intolerance or peripheral tremor.
Severe depression can present with weight loss and anxiety, however, the absence of any history suggestive of low mood and the change in bowel habit make this diagnosis as the cause of weight loss unlikely.
Alcoholism is a possibility, it can result in weight loss, change in bowel habit and tremor, however, heat intolerance would not be typical.
A 58-year-old woman goes to the endocrine clinic with a 6-month history of hot flushes and increased anxiety.
Which of the following is the most likely diagnosis?
A) Primary hyperthyroidism
Primary hyperthyroidism involves an excessive production of T3 and T4 by the thyroid gland as a result of pathology within the thyroid gland itself. The pathophysiology is as follows:
1. The thyroid produces excessive amounts of T4 and T3.
2. The excessive T4 and T3 cause negative feedback on the pituitary and hypothalamus, resulting in decreased production of TRH and TSH.
3. The end result is a raised T3 and T4 and a low TSH.
The other choices are incorrect because:
1. A TSH-secreting tumor would cause a raised TSH as well as raised T3 and T4, the TSH in this scenario is low.
2. In primary hypothyroidism, TSH is typically raised in response to a low T3 and T4.
3. In secondary hypothyroidism, TSH secretion is reduced resulting in low TSH, T3 and T4.
A 41-year-old lady presents to the GP after recently commencing carbimazole for Grave's disease. She mentions that she has had a worsening sore throat in the last 2 days. On your clinical assessment, you note she is pyrexial.
Which diagnosis do you need to consider and act upon urgently?
B) Agranulocytosis
Agranulocytosis (acute lowering of the white cell count) needs to be suspected and acted upon urgently.
Agranulocytosis may occur at any stage during treatment with Carbimazole and without warning. Patients are advised to immediately report symptoms of infection, such as a sore throat or fever, so that a full blood count test may be arranged. If this confirms a low neutrophil count, discontinuation of the drug leads to recovery. However, failure to report suggestive symptoms or delays in considering the possibility of immunosuppression and its testing can lead to fatalities.
Anaphylaxis typically presents with angioedema, pruritus, shortness of breath, wheeze, tachycardia and hypotension within minutes of an exposure to an allergen. There is no mention of any features of anaphylaxis in the history or examination and therefore this diagnosis is highly unlikely.
Quinsy (peritonsillar abscess) is a collection of pus beside the tonsil. It typically presents with a worsening unilateral sore throat and pain during swallowing. It can also feature a fever. This scenario is asking which of the options you need to consider and act upon urgently and although quinsy could be a possibility, agranulocytosis is something that needs to be considered and acted upon more rapidly given the patient is on carbimazole.
Candidiasis affecting the oesophagus can result in painful swallowing, nausea and vomiting. Although the diagnosis is a possibility it is not the most urgent diagnosis to consider in the context of a patient on carbimazole.
A 35-year-old lady with a past medical history of type 1 diabetes mellitus and Hashimoto’s thyroiditis presents to ED with fatigue, nausea and decreased appetite over the past 3 days. Examination is unremarkable apart from postural hypotension. You suspect Addison’s disease.
Which of the following electrolyte abnormalities are most in keeping with a diagnosis of Addison’s disease?
A) Hyperkalemia; hyponatremia
This is a typical presentation of Addison's disease. Addison's disease involves the destruction of the adrenal cortex, resulting in glucocorticoids (cortisol) and/or mineralocorticoids (aldosterone) deficiency. The most common cause of Addison's disease in the developed world is autoimmune-mediated destruction of the adrenal cortex. The background of two other autoimmune conditions in this patient (T1DM & Hashimoto's thyroiditis) increases the patient's risk of developing further autoimmune diseases.
Addison's disease can present with a wide range of clinical features including hypotension, fatigue, weakness, anorexia, nausea, vomiting, weight loss, non-specific abdominal pain, diarrhea, constipation, dizziness and confusion.
Clinical signs of Addison's disease include hyperpigmentation (particularly the buccal mucosa) and postural hypotension.
The most common electrolyte abnormality associated with Addison's disease is hyperkalemia and hyponatremia.
Which protein, located on the apical membrane of the follicular cells of the thyroid gland, catalyses the addition of iodine atoms to the structure of thyroid hormones during their synthesis?
C) Thyroid peroxidase
Thyroid peroxidase is an enzyme located on the apical membrane of the follicular cells of the thyroid gland that catalyses the addition of iodine atoms to the structure of thyroid hormones during their synthesis.
The fact that thyroid peroxidase is a protein on the apical membrane of follicular cells explains why antibodies against thyroid peroxidase (anti-thyroid peroxidase antibodies) are associated with an autoimmune form of hypothyroidism known as Hashimoto's thyroiditis in which the gland is destroyed by an immune response.
Thyroglobulin is the substrate of thyroid peroxidase. It is the protein precursor of the thyroid hormones to which thyroid peroxidase adds iodine atoms.
Thyroxine-binding globulin is a plasma protein that transports most of the thyroid hormone secreted by the thyroid gland throughout the circulation.
Transthyretin is another plasma protein that transports thyroid hormone throughout the circulation, but a much smaller proportion than thyroxine-binding globulin does.
Iodothyronine deiodinase I is an enzyme that converts the less biologically active form of thyroid hormone (T4) to the more biologically active form (T3) by removing an iodine atom from the structure of T4.
The activation of which receptor found on the plasma membrane of the follicular cells of the thyroid gland leads to the production of thyroid hormones and growth of follicular cells?
A) Thyroid-stimulating hormone receptor
The thyroid-stimulating hormone receptor found on the plasma membrane of the follicular cells of the thyroid gland leads to thyroid hormone production and follicular cell growth when activated by its ligand (thyroid-stimulating hormone produced by the anterior pituitary gland).
Knowing this explains why antibodies that bind to and activate the thyroid-stimulating hormone receptor are associated with an autoimmune form of hyperthyroidism known as Graves' disease, as well as why one of the clinical signs of Graves' disease is an enlarged thyroid gland (a condition known as goitre).
The thyrotropin-releasing hormone receptor is found on the plasma membrane of the cells of the anterior pituitary gland that produce thyroid-stimulating hormone, known as thyrotropes. When the ligand of this receptor (thyrotropin-releasing hormone produced by the hypothalamus) activates it, thyrotropes release thyroid-stimulating hormone.
The thyroid hormone receptor and triiodothyronine (T3) receptor are found in the nucleus of cells that can respond to thyroid hormone. Activation of this receptor by the biologically active form of thyroid hormone (triiodothyronine or T3) brings about the effects of thyroid hormone.
During which of the following sleep-related situations is growth hormone secretion by the anterior pituitary gland highest?
B) Non-rapid eye movement sleep
Growth hormone secretion by the anterior pituitary gland is highest during non-rapid eye movement sleep. Growth hormone is secreted in a pulsatile manner and there is a peak in its secretion during this stage of sleep. This may explain why sleep promotes healing and growth as well as why adolescents have an increased need for sleep compared to adults.
Rapid eye movement sleep, desynchronized sleep and paradoxical sleep are all synonyms for the same stage of sleep, during which there is less growth hormone secretion than during non-rapid eye movement sleep.
There is no peak in growth hormone secretion on awakening either.
At which stage in the uterine cycle does the menstrual phase occur?
B) Days 1-5
The menstrual phase occurs at the very beginning of the uterine cycle. It only occurs if no ovum is fertilised. The process begins with the breakdown of the corpus luteum which therefore ceases to produce progesterone. The loss of progesterone causes vasoconstriction of the spiral arteries supplying the functional layer of the endometrium. As a result, this layer becomes necrotic and is then lost through the vagina causing 3-5 days of menstrual bleeding.
Which protein produced in the follicular cells of the thyroid gland is the precursor to the thyroid hormones?
B) Thyroglobulin
The protein thyroglobulin produced in the follicular cells of the thyroid gland is the precursor to the thyroid hormones. The fact that thyroglobulin is a protein in follicular cells explains why antibodies against thyroglobulin (anti-thyroglobulin antibodies) are associated with an autoimmune form of hypothyroidism known as Hashimoto's thyroiditis in which the gland is destroyed by an immune response.
Although all of the other options given are proteins, none of them are the precursor to the thyroid hormones.
Thyroid peroxidase is an enzyme located on the apical membrane of the follicular cells that catalyses the addition of iodine atoms to thyroglobulin.
Thyroxine-binding globulin is a plasma protein that transports most of the thyroid hormone released by the thyroid throughout the circulation.
Transthyretin is another plasma protein that also transports thyroid hormone throughout the circulation, but it carries a much smaller proportion of secreted thyroid hormone than thyroid-binding globulin.
Albumin is the most abundant plasma protein in humans and is another carrier of thyroid hormone in the circulation. It carries about the same proportion of hormone as transthyretin.
The release of which of the following hormones from the hypothalamus leads to prolactin release by the anterior pituitary gland?
B) Thyrotropin-releasing hormone
The release of thyrotropin-releasing hormone from the hypothalamus leads to prolactin release by the anterior pituitary gland. This explains why thyroid disease (either hypothyroidism or hyperthyroidism) can lead to subfertilitybecause it can increase circulating prolactin levels, which has a negative effect on fertility in both females and males.
Although all of the other options are hormones released from the hypothalamus, none of them increase prolactin release by the anterior pituitary gland.
Dopamine has the opposite effect to thyrotropin-releasing hormone on prolactin release by the anterior pituitary gland; it inhibits prolactin release. None of the remaining options influence prolactin release.
During which of the following stages of life is the mean level of circulating growth hormone highest?
D) Adolescence
Mean circulating levels of growth hormone are highest in adolescence. This accounts for the pubertal growth spurtas well as growth failure during adolescence in those with growth hormone deficiency. It also explains why synthetic growth hormone can be given to children with growth hormone deficiency to induce pubertal growth.
Circulating growth hormone levels during the other options given are lower than in adolescence.
What type of glucose transporter is inserted into plasma membranes in response to insulin?
C) GLUT4
GLUT4 is inserted into plasma membranes in response to insulin. It is responsible for insulin-regulated glucose storage and is mainly found in adipose tissues and striated muscle.
GLUT2 is mainly found in renal tubular cells, liver cells, the basolateral membrane of the small intestine epithelium and pancreatic beta cells.
GLUT3 is mainly expressed in neurons and the placenta.
GLUT1 is widely found in fetal tissues, erythrocytes, as well as in the blood-brain barrier.
GLUT14 is mainly found in testicular optic canals.
Which of the following hormones surges prior to ovulation?
B) Luteinizing hormone
During the follicular phase, a rise in FSH from the pituitary stimulates the development of several follicles on the surface of the ovary. Each follicle contains an egg. Later, as the FSH level decreases, only one follicle continues to develop. This follicle also produces estrogen.
The LH peaks mid-cycle (known as the LH surge), triggering the release of the ovum (known as ovulation), which usually occurs 16-32 hours after the surge begins. The LH level falls a couple of days later.
The estrogen level from the ovaries increases gradually towards ovulation and peaks during the LH surge.
Which of the following glands is the only gland in the human body known to require iodine to function?
E) Thyroid
The thyroid gland is the only gland in the human body known to require iodine to function. Iodine obtained in the diet is needed to produce the hormones made by the thyroid gland, which are tetraiodothyronine (also known as thyroxine) and triiodothyronine. The abbreviated names of these hormones (T4 for tetraiodothyronine and T3 for triiodothyronine) are derived from the number of iodine atoms in the structure of each of these hormones; 4 in tetraiodothyronine and 3 in triiodothyronine. Enzymes known as iodothyronine deiodinases convert the less biologically active form of thyroid hormone (T4) into the more biologically active form (T3) by removing an iodine atom from the structure of T4.
Knowing that the thyroid gland requires iodine to function explains why a deficiency of iodine in the diet leads to an underactive thyroid gland (a condition known as hypothyroidism) as well as why dietary iodine deficiency causes the thyroid gland to enlarge (a condition known as goitre) because the lack of T4 and T3 produced by the thyroid gland due to iodine deficiency means that these hormones do not inhibit the release of thyroid-stimulating hormone from the pituitary gland as they usually do when produced. As a result, this unopposed increase in thyroid-stimulating hormone causes the thyroid gland to enlarge.
None of the other glands mentioned require iodine to function. The pineal gland produces melatonin. The pituitary gland produces many hormones that control the actions of other endocrine glands in the body, including adrenocorticotrophic hormone, thyroid-stimulating hormone, growth hormone, prolactin, follicle-stimulating hormone, luteinising hormone, melanocyte-stimulating hormone, oxytocin and antidiuretic hormone. The parathyroid glands produce parathyroid hormone. The adrenal gland produces aldosterone, cortisol, androgens and adrenaline.
Which of the following mechanisms leads to the stimulation of aldosterone?
A) Increase in plasma potassium
An increase in plasma potassium concentration results in the stimulation of aldosterone. Aldosterone increases sodium, and therefore water, reabsorption and promotes the secretion of potassium into the tubular lumen. It achieves this by up-regulating Na/K pumps in kidney nephrons. Aldosterone primarily acts upon the distal tubules and collecting ducts of nephrons.
Other factors that stimulate aldosterone release include:
1. plasma acidosis
2. increase in angiotensin II
3. increase in ACTH
Pulsing of which hormone first precipitates puberty?
D) Gonadotropin-releasing hormone
The onset of puberty is associated with high GnRH pulsing, which precedes the rise in sex hormones, LH and FSH. Exogenous GnRH pulses cause the onset of puberty. Brain tumours which increase GnRH output may also lead to premature puberty.
What hormone induces the milk-down reflex?
D) Oxytocin
Oxytocin induces the milk-down reflex (also known as the let-down reflex and milk-ejection reflex). This peptide hormone is produced in the paraventricular nucleus of the hypothalamus and released from the posterior pituitary in response to suckling. In order to achieve the milk-down reflex, oxytocin causes the contraction of band-like smooth muscle cells surrounding alveoli in the breast containing milk. This squeezes milk into the breast ductal system. Oxytocin is also responsible for causing the contraction of uterine smooth muscle during and after birth. Prolactin is also involved in the process of lactation, but instead is responsible for the synthesis of breast milk rather than its expulsion.
What type of tissue does the posterior pituitary mainly consist of?
A) Neural
The posterior pituitary gland mainly comprises of neural tissue. Despite being a component of the endocrine system, this structure is essentially a collection of axons extending from the supraoptic and paraventricular nuclei of the hypothalamus. Peptide hormones are initially released by these axons into the capillaries of the hypophyseal portal circulation. These hormones are then stored in Herring bodies prior to being secreted by the posterior pituitary gland.
Gondatrophin-releasing hormone released by neurons in the hypothalamus causes follicle-stimulating hormone and luteinising hormone to be released from the cells in the anterior pituitary gland. Some people are born without these neurons, a condition known as Kallman's syndrome, and, as a result, do not undergo puberty and exhibit subfertility.
A) Both statements are true.
Which of the following proteins is the precursor to the hormone adrenocorticotrophic hormone produced by corticotrophs in the anterior pituitary gland?
D) Proopiomelanocortin
Proopiomelanocortin (POMC) is the precursor to adrenocorticotrophic hormone (ACTH) produced by corticotrophs in the anterior pituitary gland. POMC yields different products depending on how it is cleaved by different enzymes. Beta-lipotropin is one such product of initial cleavage of proopiomelanocortin, but its role in human physiology is unclear. Melanocyte-stimulating hormone (MSH), which leads to increased melanin production by melanocytes, is another product of cleavage of POMC, specifically cleavage of a fragment containing the sequence for both ACTH and MSH, but most of these POMC fragments in humans are converted into ACTH and only a small number into MSH. However, when there is an excess of ACTH, e.g. if there is a tumour secreting ACTH or if there is an excess of ACTH due to lack of negative feedback on the neurons in the hypothalamus releasing corticotropin-releasing hormone as a result of destruction of the adrenal gland (a condition known as Addison's disease), there is an excess of the POMC fragment from which both ACTH and MSH are produced and so more MSH is produced from POMC than usual.
This explains why one of the signs of Addison's disease is darkening of the skin, especially in the oral cavity and palmar and plantar creases.
The other two options are not produced by corticotrophs in the anterior pituitary gland.
Corticotropin-releasing hormone is produced by the hypothalamus and triggers ACTH release from corticotrophs in the anterior pituitary gland.
Cortisol is a hormone released from the adrenal cortex in response to ACTH.
Which glucose transporter is associated with 'glucose sensing' in the liver/pancreas?
A) GLUT2
GLUT2: Present in the liver + pancreas; allows glucose sensing; functions as a bidirectional transporter, glycolysis, glycogenesis and gluconeogenesis; present in the beta-cells of the pancreas; transports all 3 monosaccharides
GLUT1: Basal glucose uptake, mostly in the brain, specifically around the blood-brain barrier; expression of GLUT1 is upregulated in many tumors
GLUT3: Basal glucose uptake, mostly in the brain, specifically expressed in neurones (axons and dendrites); highest affinity for glucose, works very well in low glucose concentrations; necessary for healthy neuronal function.
GLUT4: Present in muscle, heart and adipose tissue; responds to insulin, 'insulin-regulated'; stored intracellularly in transport vesicles, incorporates into plasma membrane when insulin binds to its membrane receptor
Which of the following structures produce gonadotropins?
B) Anterior pituitary
The anterior pituitary is involved in the production and storage of several major hormones including gonadotropins. Therefore, this is the correct answer. Others include adrenocorticotrophic hormone (ACTH), thyroid stimulating hormone (TSH), prolactin, growth hormone and melanocyte-stimulating hormone.
The other options are incorrect and their functions are described below.
1. The hypothalamus is found at the base of the brain and is involved in controlling metabolic processes and parts of the autonomic nervous system. It is involved in the production and secretion of neurohormones, hypothalamic releasing hormones, that regulate the secretion of pituitary hormones. However, it does not produce gonadotropins, therefore, is not the correct answer.
2. The ovaries work to produce oocytes and reproductive hormones (estrogen and progesterone) involved in normal physiology and pregnancy. However, the ovaries do not produce gonadotropins, therefore this is not the correct answer.
3. The main function of the endometrium is to line the uterus and is the site of implantation. It also prevents the uterine cavity from collapsing. This is not the correct answer as the endometrium does not produce hormones.
Which days of the menstrual cycle have the highest probability of fertilization?
A) 12-14
The LH surge, which occurs on day 12-14 of the menstrual cycle, occurs 24 to 48 hours prior to ovulation, resulting in the release of the ovum from the Graafian follicle, therefore resulting in the highest probably of fertilisation.
The Graafian follicle then becomes the corpus luteum and secretes progesterone (luteal phase) which causes the thickening of the cervical mucus and decreases the probability of sperm entry and therefore fertilization.
All of the following hormones are produced in the anterior pituitary gland, except:
F) Alpha-endorphin
It should be beta-endorphin.
Which one of the following cells types does not produce estrogen?
C) Theca cells
Theca cells do not produce estrogen. When stimulated by luteinizing hormone, theca cells produce progesterone and also androstenedione. Androstenedione is a precursor molecule in the synthesis of estrogen.
In the ovary, the androstenedione produced by theca cells is taken up by granulosa cells. These cells contain aromatase (and other enzymes), which convert androstenedione into testosterone and then estrogen.
During pregnancy, the cytotrophoblasts of the placenta produce many essential hormones including oestrogen, human placental lactogen and human chorionic gonadotrophin.
Interestingly the human gonadal cells have been found to produce significant amounts of oestrogen. Indeed, aromatase has been found in Sertoli, Leydig and even in male germ cells, suggesting that they are capable of producing estrogen. The role of estrogen in the male reproductive system is debated, but it is possible that they are required for sperm motility.
Fat cells also contain aromatase, which means that they can also produce estrogen. In fact, there are many sources of estrogen in peripheral tissue, including the liver and brain, and these sources are more important in post-menopausal women.
In a normal 28 day menstrual cycle, when would you expect the luteinizing hormone (LH) surge to occur?
D) Day 11-13
The LH surge usually occurs on day 12.
This occurs due to oestrogen levels reaching a threshold level which triggers large amounts of LH production. The high LH levels cause the membrane of the Graafian follicle to become thinner. As a result, 24-48 hours after the LH surge ovulation usually occurs due to the rupture of the Graafian follicle.
When growth hormone is released from the anterior pituitary gland and activates its receptor on target cells, the production of which of the following molecules, which mediates most of the effects of growth hormone, will be stimulated in these cells?
D) Insulin-like growth factor-1
When growth hormone is released from the anterior pituitary gland and activates its receptor on target cells, the production of insulin-like growth factor-1, which mediates most of the effects of growth hormone, is stimulated in these cells. Insulin-like growth factor-1 is largely responsible for the anabolic effect of growth hormone, i.e. bone and muscle growth. Growth hormone also increases blood glucose by stimulating gluconeogenesis in the liver.
This explains why children with insulin-like growth factor-1 deficiency experience growth failure and why synthetic insulin-like growth factor-1 can be used to treat this.
Cortisol, like growth hormone, increases blood glucose, but is produced by the adrenal cortex, not the anterior pituitary gland, and does not stimulate insulin-like growth factor-1 production in target cells.
Glucagon, like cortisol and growth hormone, also increases blood glucose, but is produced by the alpha cells of the pancreas, not the anterior pituitary gland, and does not stimulate insulin-like growth factor-1 production in target cells.
Insulin, although producing anabolic effects like growth hormone, is produced by the beta cells of the pancreas, not the anterior pituitary gland, and does not stimulate insulin-like growth factor-1 production in target cells. Furthermore, insulin reduces blood sugar, whereas growth hormone increases it.
Although somatostatin is produced in response to growth hormone, it is produced in the hypothalamus, not the anterior pituitary gland. This is a negative feedback mechanism of growth hormone that limits its production because somatostatin inhibits growth hormone production by the anterior pituitary gland. Somatostatin also does not induce insulin-like growth factor-1 production in its target cells.
What mechanism is used to transport iodide into thyroid follicular cells?
D) A sodium-iodide co-transporter
Iodide is transported into thyroid follicular cells by a sodium-iodide cotransporter (also known as a sodium-iodide symporter).
The transport of iodide into thyroid follicular cells is pivotal for the synthesis of iodine and, subsequently, thyroid hormones. Sodium-iodide cotransporters actively transport iodide into follicular cells via the basolateral membrane. Chloride-iodide cotransporters (called pendrin) then transport iodide into the colloid via the apical membrane of follicular cells.
Which of the following effects of growth hormone leads to an increase in height during adolescence?
D) Stimulating bone formation by osteoblasts
Stimulating bone formation by osteoblasts is the mechanism by which growth hormone leads to an increase in height during adolescence.
This explains why an excess of growth hormone before bones stop growing vertically, i.e. before the closure of the epiphyseal growth plates, causes gigantism.
Although all of the other options are effects of growth hormone, none of them account for the increase in height during adolescence. In fact, inducing somatostatin release from the hypothalamus is a negative feedback mechanism of growth hormone that limits its own secretion because somatostatin inhibits growth hormone release from the anterior pituitary gland.
Around which of the following times is glucocorticoid secretion lowest?
E) 00:00
Glucocorticoid secretion is lowest around 00:00. Glucocorticoid levels are different at night compared to the day (a pattern known as a diurnal cycle).
This explains why one of the tests for Cushing's syndrome, a condition in which there are excess circulating glucocorticoids, is a blood test to measure cortisol levels at midnight.
None of the other options constitute the time around which glucocorticoid levels are lowest.
Which of the following represents the function of thyroid peroxidase?
B) Iodinates thyroglobulin
Thyroid peroxidase is a multifunctional enzyme, which has three main functions. Thyroid peroxidase oxidizes molecular iodine, catalyzes the attachment of iodine to tyrosine residues on thyroglobulin and also catalyzes the coupling of two iodinated tyrosine residues. Thus, its function is essential to the production of thyroid hormone.
The enzyme that converts T4 to T3 in cells in 5'/3' monodeiodinase.
Which cells secrete testosterone in the testes?
B) Leydig cells
Leydig cells are found outside the seminiferous tubules of the testes. These cells produce testosterone as a result of LH (luteinizing hormone) secretion from the pituitary gland.
Sertoli cells are part of the seminiferous tubules of the testes. These cells maintain a favorable environment for developing sperm cells which is essential for sperm production. The hormone FSH (follicle-stimulating hormone) activates the Sertoli cells.
Myoid cells are squamous contractile cells which produce peristaltic waves, surrounding the seminiferous tubules of the testis.
Spermatogonia stem cells function is to produce sperm from undifferentiated germ cells in the seminiferous tubules of the testis.
Which of the following hormones triggers the development of the dominant follicle?
E) Follicle-stimulating hormone
Follicle stimulating hormone (FSH) triggers the development of many follicles, one of which develops into the dominant follicle. This dominant follicle or Graafian follicle will eventually release an egg during ovulation ready for fertilization.
The other options are incorrect for the following reasons.
1. Luteinizing hormone (LH) acts to trigger ovulation, maintains the corpus luteum and controls the secretion of progesterone.
2. Beta-human chorionic gonadotropin (B-hCG) is a placental hormone produced by the syncytiotrophoblast cells that maintains the corpus luteum, which produces progesterone that can support the endometrial lining and maintains the pregnancy.
3. Prolactin (PRL), produced by the anterior pituitary gland, promotes lactation in women after childbirth and is not involved in the development of the dominant follicle.
4. Gonadotropin-releasing hormone (GnRH) indirectly stimulates the development of the dominant follicle by stimulating the release of FSH from the anterior pituitary gland but it is not directly responsible and so is not the best answer.
Which of the following is a hormone that inhibits the release of growth hormone from the anterior pituitary gland?
D) Somatostatin
Somatostatin is a hormone released by neurons in the hypothalamus that inhibits the release of growth hormone from the cells in the anterior pituitary gland that produce growth hormone, known as somatotrophs. This explains why synthetic somatostatin is used to treat acromegaly, a condition whereby there is an excess of circulating growth hormone.
Rosuvastatin and simvastatin are not hormones. They are both lipid-lowering drugs of the statin class.
Both follistatin and inhibin are molecules produced by the ovaries in females that inhibit follicle-stimulating hormone secretion from the anterior pituitary gland.
Which of the following hormones does not lead to an increase in plasma glucose levels in response to hypoglycemia?
A) Testosterone
Testosterone does not affect blood glucose levels, instead, it has several other functions including development of male reproductive tissues, development of secondary sexual characteristics (including increased muscle and bone mass) and the growth of body hair.
Why does the hormone hCG mimic the action of LH in the preservation of the corpus luteum post-ovulation?
B) They are both glycoproteins, and thus share a common alpha unit.
hCG and LH are both glycoprotein hormones. They share a common alpha subunit, and their own unique beta unit (which determines their function). Since they share a common subunit they are able to act on the same receptors, in this case, the LH receptors on the corpus luteum.
Which of the following is an effect of growth hormone that allows it to act as a counterregulatory hormone to insulin, meaning that it opposes the effect of insulin?
B) Increasing blood glucose levels by increasing gluconeogenesis by the liver
Growth hormone acts as a counterregulatory hormone to insulin by increasing blood glucose levels by increasing gluconeogenesis in the liver. Growth hormone increases blood glucose levels, whereas insulin reduces blood glucose levels.
This explains why giving synthetic growth hormone to patients can increase their blood glucose levels.
None of the other options are counterregulatory effects of growth hormone.
Reducing blood glucose levels by reducing gluconeogenesis by the liver is the exact opposite of the counterregulatory effect of growth hormone.
Although glycogenolysis by the liver increases blood glucose levels, increasing blood glucose levels by increasing glycogenolysis by the liver is not how growth hormone increases blood glucose levels. It does so by increasing hepatic gluconeogenesis.
Reducing blood glucose levels by increasing glycogen synthesis by the liver is an effect of insulin, not growth hormone.
Increasing blood glucose levels by reducing gluconeogenesis by the liver is wrong because reducing hepatic gluconeogenesis reduces blood glucose levels rather than increasing them. Growth hormone also increases rather than reduces hepatic gluconeogenesis.
What is the function of the incretin hormones?
A) Stimulates decrease in blood glucose levels
Incretin hormones act to decrease blood glucose levels by increasing the release of insulin from beta cells.
Insulin is released from pancreatic beta cells when glucose levels are increased. Insulin acts to decrease these raised glucose levels to bring blood glucose levels back to the norm. The pathway of insulin release is as follows: glucose enters pancreatic beta cells (via GLUT 2 transporter), glucose is then is utilised by the cell to increase ATP production, this ATP causes the potassium channel to close, which leads to depolarisation of the beta cell, this triggers the opening of voltage-gated calcium channels, calcium moves in and causes release of insulin from secretory vesicles.
Incretins act in this pathway to increase the release of insulin. Incretins are released from the gastrointestinal tract in response to food consumption. The incretin hormones act on receptors on beta cells to amplify insulin release. They also act to decrease the release of glucagon.
The two main incretin hormones in humans are GIP (gastric inhibitory peptide) and GLP-1 (glucagon-like-peptide-1).
This physiology is relevant clinically because incretins can be targeted by DPP-4 inhibitors such as sitagliptin which increase incretin levels, helping to control blood glucose in type 2 diabetes.
Around which of the following times is glucocorticoid secretion the highest?
B) 08:00
Glucocorticoid secretion is highest around 08:00. Levels of glucocorticoids are different at night compared to the day (a pattern known as a diurnal cycle).
This is important to bear in mind when interpreting blood tests used to measure glucocorticoids performed on samples taken around this time. Furthermore, if the person was stressed when the blood samples were taken, stress-induced glucocorticoid secretion will elevate levels even further.
None of the other options constitutes the time around which glucocorticoid secretion is highest.
Which tissue synthesizes and secretes IGF-1 in response to growth hormone?
C) Liver
The liver synthesizes and secretes IGF-1 in response to growth hormone released by the anterior pituitary.
IGF-1 (also known as somatomedin C) plays a pivotal role in promoting growth throughout childhood. This protein is also involved in the regulation of DNA synthesis.
Clinical significance
1. Laron dwarfism: a condition that results from a lack of growth hormone receptors so IGF-1 cannot be produced; growth hormone levels are typically normal
2. Acromegaly: a condition that results from an excessive release of growth hormone (typically due to the presence of a pituitary adenoma) and, subsequently, IGF-1.
When the catecholamines norepinephrine and epinephrine are released from the adrenal medulla, their activation of which of the following adrenoreceptors leads to an increase in blood pressure?
C) Alpha-1
The activation of alpha-1 adrenoreceptors by catecholamines released from the adrenal medulla leads to increased blood pressure by causing vasoconstriction.
This explains why drugs that block alpha adrenoreceptors can be used in the pharmacological management of a rare catecholamine-secreting tumour of the adrenal medulla known as phaeochromocytoma, one of the signs of which is treatment-resistant hypertension.
When activated, alpha-2 adrenoreceptors actually oppose the effect of alpha-1 adrenoreceptors, i.e. they reduce blood pressure by causing vasodilation.
Alpha-3 adrenoreceptors is wrong because it is a made-up option.
Beta-2 adrenoreceptors mediate many physiological effects of catecholamines, but mainly relaxation of visceral smooth muscle.
Beta-3 adrenoreceptors mainly mediate lipolysis.
Withdrawal of which of the following hormones causes menstruation at the end of the menstrual cycle?
D) Progesterone
If fertilization does not occur post-ovulation, reduced levels of LH and FSH (and no production of hcG by the decidua) cause progesterone production from the corpus luteum to decrease. This triggers spiral arteries in the uterus to constrict, causing ischaemia and consequent necrosis of the functional layer of the endometrium, which is then shed.
All of the following will result to elevated ACTH levels, except:
C) Long-term glucocorticoid therapy
If the concentration of glucocorticoids, the hormones produced by the adrenal cortex in response to adrenocorticotrophic hormone, in the blood rises, this will lead to reduced adrenocorticotrophic hormonesecretion by the anterior pituitary gland. Since adrenocorticotrophic hormone causes the adrenal glands to grow, negative feedback by glucocorticoids will cause them to reduce in size.
What lobes can be seen in the thyroid gland?
D) Left, right pyramidal
The thyroid gland is made up of the left, right and pyramidal lobes.
While most thyroids contain only left and right lobes, the pyramidal lobe is a common and recognised variant of the normal thyroid. The estimated prevalence is up to 45%. This lobe projects superiorly off the isthmus of the thyroid gland and is shaped like an isosceles triangle.
The pyramidal lobe is thought to be a remnant of the embryological thyroid as it descended through foramen caecum at the dorsal tongue, and continued down the anterior surface of the thyroid and cricoid cartilage.
The other answers available are not recognised names for the thyroid gland lobes.
At what point anatomical point would you expect to find the isthmus of the thyroid gland?
A) Halfway between the thyroid cartilage and the sternal notch
The isthmus of the thyroid gland is found halfway between the thyroid cartilage and the sternal notch.
The upper border of thyroid cartilage (C3/C4 level) is the level at which the carotid bifurcation occurs.
The oesophagus narrows in the cervical region at the level of cricopharyngeus (C5/C6 level).
The hyoid bone is found at the base of the mandible (C3 level).
The vertebral arteries enter the transverse process of C6 or C7 and pass superiorly in the transverse foramen of each cervical vertebra.